The NLCS Jeju Journal of Pure and Applied Mathematics Volume 5, April 2025

Page 1


Volume5,Issue1

MathematicsSociety MathematicsPublicationCCA

April2025

Credits

Firstofall,wewouldliketoexpressoursinceregratitudetothefollowingcontributorstoVolume5Issue1 ofNLCSJejuAnnualJournalofMathematics.

TheMathematics Society

Chair

JeffSuhyukCho(12)

DerekYejunYoo(12)

PublicityOfficer

SoulPark(11)

Secretary JaydenJunseokLee(11)

Y12Leaders

AustinTaehongHa(12)

SeanTaehoonKim(12)

LinkTeachers

MsDuyguBulut

MrWilliamHebbron

LATEXEditors&Managers

JeffSuhyukCho(12)

DerekYejunYoo(12)

RyanHanjinLee(12)

JaydenJunseokLee(11)

AustinTaehongHa(12)

SeanTaehoonKim(12)

SoulPark(11)

ElliottHyunseokLee(12)

Wewouldalsoliketoexpressoursinceregratitudetoourlong-servingformerHeadoftheMathsDepartment, Ms.Bulut,whoserelentlesssupportandenthusiasmhavebeenaconstantsourceofinspiration.Thisjournal wouldnothavebeenpossiblewithouther.Wishingyouallthebestonyournextchapter,Ms.Bulut.

© CopyrightbytheNLCSJejuJournalofPureandAppliedMathematics EditedbytheJPAMEditingTeam

Editor’sNote

WelcometoanotherremarkableeditionofJPAM,acollectionofrigorousmathematicalarticleswrittenbythebrilliantmindsofNLCSJeju!This year’sselectionfeaturessomeofthemostintriguingandthought-provoking topicstheMathsSocietyhaseverexplored.Butdon’tbeintimidated: whetherit’stheRiemannzetafunctionoranotherabstractconcept,you aresuretofindsomethingfascinatingwithinthesepages.Andifsomearticlesseemtoochallengingnow,revisittheminafewyears—youmightjust discoveryournewfavouritepieceofmathematics!

JPAMliesattheheartoftheMathsSociety,acherishedtraditionpassed downthroughtheyearsandonethatwillundoubtedlycontinueinthefuture. Whileaspectsofthejournalhaveevolved,likeoursleeknewcoverdesign, itscoremissionremainsunchanged:tosharethevaluableinsightsofour school’smathematiciansandtoignitecuriosityinallreaders,regardlessof theirinterests,backgrounds,orexpertise.Thisjournalisnotaboutshowcasingability;rather,itexistsforanyoneeagertoexpandtheirknowledge, eventotheslightestdegree.

Asyouread,keepthisinmind:ifyougraspaconcept,fantastic!Ifnot, simplymoveontothenextarticle.Mathematics,likeallsubjects,isajourney.Understandingdoesn’talwayscomeinstantly.Thetopicscoveredhere areindeedadvanced,extendingbeyondthestandardcurriculum,butifeven asinglearticlebroadensyourperspective,wewillhaveaccomplishedourgoal.

Pleaseenjoy.

VietaJumping

Doyeon(Alex)Kim Year10

Email:dykim28@pupils.nlcsjeju.kr

Reviewer

Hanjin(Ryan)Lee∗

RecommendedYearLevel:KS5

Keywords:NumberTheory

1Introduction 1988IMOP6

Letaandbbepositiveintegerssuchthatab+1divides a2 + b2.Showthat a2+b2 ab+1 isasquareofaninteger.

ThequestionaboveisthelastquestionfromIMO(InternationalMathematicalOlympiad)1988.Nobodyofthesix membersoftheAustralianproblemcommitteecouldsolve it.Sinceitwasanumbertheoreticproblemitwassenttothe fourmostrenownedAustraliannumbertheorists.Theywere askedtoworkonitforsixhours.Noneofthemcouldsolve itintime.Theproblemcommitteesubmittedittothejuryof theXXIXIMOmarkedwithadoubleasterisk,possiblytoo hardtopose.Afteralongdiscussion,thejuryfinallyhadthe couragetochooseitasthelastproblemofthecompetition. Elevenstudentsgaveperfectsolutions.Then,whatwasthe perfectsolution?Itusesanumbertheoretictechniquecalled “VietaJumping”orinotherwords,“RootFlipping”.

2WhatisVietaJumping?

VietajumpingisusedtosolveDiophantineEquations (anindeterminateequationwithintegerroots),inequalities orcongruenceswherethesolutionshavearecursivestructure.

ItreliesonVieta’sformulasfortherootsofaquadratic equationandinvolvessystematically”jumping”fromonesolutiontoanother.VietaJumpingisnotatheoremlikeFermat’slittletheoremorStewart’stheorem.Itisatechnique usedtoprovepropositions.

1.Startwiththegivenequation:Typically,theproblem involvesanintegerequation,oftensymmetricalinsome way

2.Assumeaninitialsolution:Findorassumeoneintegersolution(oftenthesmallestpositivesolution)tothe givenequation.

3.Formaquadraticequation:Usethegivenequationto constructaquadraticequationwhereonevariable(x)is treatedasarootwhiletheother(sayy)istreatedasa parameter.

4.ApplyVieta’sFormula:Asyoualreadyknowonesolutionwhichwouldbetheintegersolutionyouwouldalso knowtheothersolutionwhichwouldalsobeaninteger.

5.Jumptoanewsolution:Usingthepropertiesofthe quadraticequation,you”jump”toanothersolutionby replacingoneroot(x)withtheother(x’),whichisguaranteedtobeanintegerifthecoefficientsareintegers.

6.Iterateandfindsmallersolutions:Byrepeatedlyjumping,youeitherreducetheproblemtoasimpleroneor reachacontradiction,provingsomethingaboutthenatureofthesolutions(e.g.,minimality).

7.Concludetheproof:Theprocessoftenprovesastatementbyshowingthatnosmallerpositivesolutionexists, oritarrivesatacontradiction.

AsVietajumpingisnotatheorem,ratheraprooftechnique,showinganexamplewouldbeamoreeffectiveway ofexplainingit.

31988IMOExample(ClassicalVietaJumping) 1988IMOP6

Letaandbbepositiveintegerssuchthatab+1divides a2 + b2.Showthat a2+b2 ab+1 isasquareofaninteger.

Assumethat a2+b2 ab+1 = k(k ∈ N wherekisnotaperfect square.Fixkandletthesetofpositiveintegersolutions (a,b)thatsatisfy a2+b2 ab+1 = k betset S.Duetowell-ordering principle,inset S,therewillexist(a,b)s.t.a+bisminimised. Letthissetof(a,b)be(A,B).W.L.O.G. A ≥ B > 0

Denotethetworootsof 1 ⃝ by x1 and x2 1 ⃝ willahvean integerrootAandifweapplyVieta’sformula,

x1 + x2 = kB

x1x2 = B2 k

W.L.O.G.Let x1 = A.Ifso, x2 = kB A = B2 k A

As kB A isaninteger, B2 k A ∈ Z

IF x2 = 0,then k = B2 −→ contradictionIf x2 < 0,then x2 2 kBx2 +(B2 k)= x2 2 + B2 k(Bx2 + 1) > 0 −→

contradiction(x2 2 kBx2 + B2 k = 0) x2 > 0 −→ (x2, B) ∈ S

x2 = B2 k A < B2 A ≤ A −→ x2 < A −→ x2 + A < A + B −→ contradictionasA+B(Minimality)Otherwise,wecancontinue, using (b, x2) insteadof (A, B).Wegetadecreasingsequence ofpositiveintegers.

[a1, a2, a3]=[a, b, b2 k a ...]

Forall n(n ∈ N),thepair (an, an + 1) willsatisfy

an 2+an+12

anan+1+1 = k.Asthissequencecannotcontinueindefinitely, atsomepointoneofthenumberswillbe0,provingthatkis aperfectsquare.

4Anotherexample

Whena,b,cisanaturalnumber,a,b,csatisfies0 < a2 + b2 abc ≤ c.Showthat a2 + b2 abc isaperfectsquare.

Let’sdenote a2 + b2 abc as k(0 < k ≤ c)

W.L.O.G.Let a ≤ b(symmetrical)

As a2 + b2 abc = k ←→

a2 + b2 = abc + k ←→

abc < a2 + b2 = abc + k ≤ ab + b2

ac < a + bb > a(c 1)

Then,therewillbelargelytwocasespossible, Case1) b > ac Case2) b ≤ ac

Case1)

If b > ac, b canbeexpressedas b = ac + t(t ∈ N)

a2 + b2 abc = ... 1 ⃝.Ifwesubstitute b = ac + t into equation 1 ⃝, a2 + a2c2 + 2act + t2 a2c2 tac = k ⇔a2 + act + t2 = k ≤ c−→ contradiction(a,t ∈ N)

Case2)

if b ≤ ac, b canbeexpressedas b = ac s(s ∈ N) a2 + b2

abc = k 1 ⃝⇔b2 ac × b +(a2 k)= 0... 2 ⃝

Let’sdenotethetworootsofthequadraticequationof 2 ⃝ as b1 and b2.IfweapplyVieta’sFormula, b1 + b2 = acb1b2 = a2 k Howeve,a s = ac b,wecanrecognise s asanotherformof b s = a2 k b < a.Cahgningtheform, a2 acs + s2 k = 0.

If s = 0, k = a2,thusaperfectsquare.

If s = 0, a2 csa + s2 k = 0.Let’sdenotethetwo rootsofthequadraticequation a1 and a2.IfweapplyVieta’s formula,

a1 + a2 = cs

a1a2 = s2 kcs a isalsoasolutiontotheequation. r = s2 k a < s.If (a, b) established,therewillexistapair (r, s) Ifweassumethat a + b isminimised,as a + b > r + s −→ contradiction(minimalism).

Therefore, k = a2.As k = a2 + b2 abc, a2 + b2 abc isa perfectsquare.

5ConstantDescentVietaJumping

TherearealsovariationsinVietaJumping.Themethod ofconstantdescentVietajumpingisaprooftechniqueoften usedtoestablishstatementsinvolvingaconstantkandthe relationshipbetweenaandb.UnliketraditionalVietajumping,thismethoddoesnotrelyoncontradictionandconsists ofthefollowingfoursteps:

1.Provethecasewhereequalityholds,allowingtheassumptionthat a > b

2.Fixthevaluesof b and k,thenrewritetherelationship between a, b,and k asaquadraticequationwithcoefficientsinvolving b and k.Onerootofthisquadraticis a, whiletheotherrootisdeterminedusingVieta’sformulas.

3.Forallpairs (a, b),beyondacertainbasecase,demonstratethat o < x2 < b < a,andthat x2 isaninteger. Thenreplace(a,b)with (b, x2),repeatingthisprocess untilreachingthebasecase.

4.Provethestatementforthebasecase.Since k remains constantthroughouttheprocess,thestatementisvalid forallorderedpairs (a, b).

Thisensuresthevalidityofthestatementforallrelevant cases.Asaforementioned,asVietaJumpingisaprooftechnique,notatheory,anexamplewillexplaintheabovemore intuitively.

Let a and b bepositiveintegerssuchthat ab divides ab + 1.

Provethat3ab = a2 + b2 + 1.

If a = b, ab|ab + 1 ⇔ a2|2a2 + 1,as a2|2a2 forall a ∈ Z, thisimpliesthat a2|1.

Hence, a and b willbothbe1and32 = 12 + 11 + 1.

So,W.L.O.G.assumethat a > b

Forany (a, b) satisfyingthegivenconditions,let a2+b2+1 ab = 3.Ifwerearrangethisandsubstitute,wecangetthefollowingquadraticDiophantineequation.

x2 (kb)x +(b2 + 1)= 0... 1 ⃝

IfweapplyVieta’sformula,

x1 + x=kb

x1x2 = b2 + 1

Asonerootofthequadraticequationisa,theotherroot, x2 = kb a = b2+1 1 ∈ Z.Thefirstequationshowsthat x2 is anintegerandthesecondthatitispositive.As a > b and because a, b ∈ Z, a ≥ b + 1andhence ab ≥ b2 + b.If b > 1, ab > b2 + b wouldalwayshold,andtherefore x2 = b2+1 a < b Thus,whilemaintainingthesame k,wemayreplace (a, b) with (b, x2) andrepeatthisprocessuntilwearriveatthebase case.

Asthebasecaseiswhen b = 1,for (a, 1) tosatisfythegiven

condition, a shoulddivide a2 + 2.Yet a|a2 forall a ∈ Z, a|2. Ifso, a shouldbeafactorof2,makinga1or2.

Thefirstcaseiseliminated −→ contradiction(a=byeta¿b)

Inthesecondcase, k = a2+b2+1 ab = 6 2 = 3. As k hasremainedconstantthroughoutthisprocessof Vietajumping,thisissufficienttoshowthatforany (a, b) satisfyingthegivencondition, k = 3.

6GeometricalInterpretationofVietaJumping Vietajumpingisconsideredtobeanumbertheorictechniquewithincludespartsofalgebra.However,Vietajumpingcanalsobeinterpretedgeometrically.Thegeneralprocedureisasfollows:

1.Usingthegivencondition,derivetheequationofafamilyofhyperbolasthatremainunchangedwhen x and y areswapped,makingthemsymmetricabouttheline y = x

2.Provethepropositionfortheintersectionsofthehyperbolasandtheline y = x

3.Assumethereissomelatticepoint (x, y) onsomehyperbolaandW.L.O.G.assumethat x < y.ThenbyVieta’s formulas,thereisacorrespondinglatticepointwiththe samex-coordinateontheotherbranchofthehyperbola, andbyreflectionthrough y = x anewpointontheoriginalbranchofthehyperbolaisobtained.

4.Showthatthisprocessgenerateslatticepointsprogressivelyloweronthesamebranchandcanberepeateduntilaspecificcondition(suchas x = 0)isreached.Substitutingthisconditionintotheequationofthehyperbola, thestatementcanthenbeproven.

Let’sgobacktothe1988IMOP6,andinterpretthe solutionintermsoflatticepointsonhyperbolasinthefirst quadrant.

1988IMOP6

Let a and b bepositiveintegerssuchthat ab + 1divides a2 + b2.Showthat a2+b2 ab+1 isasquareofaninteger.

Let a2+b2 ab+1 = q where q isnowafixedvalue.If q = 1, q isapefectsquare.if q = 2,theredoesnotexist (a, b) where a, b ∈ Z.Thus,when q > 2,theequation

x2 + y2 qxy 1 = 0

Defineshyperbola H,and(a,b)representsanintegrallatticepointon H

If (x, x) isanintegrallatticepointon H where x > 0, then x = 1as q ∈ Z

Thepropositionistrueforthepoint (x, x)

Let P =(x, y) bealatticepointon H,where x, y > 0and x = y.Duetosymmetry(oftheline y = x),assume x < y.

IfweapplyVieta’sformula, (x, qx y),isalatticepointon thelowerbranchof H.

Let y′ = qx y.From H,1 + xy′ > 0andsince x > 0, y′ ≥ 0. Hence (x, y′) isonthefirstqudrant(exceptwhen y′ = 0).Due tosymmetry, (y′ , x) isalsoon H onthefirstquadrant.

IfweapplyVieta’sformula, yy′ = x2 q,and y′ = x2 1 y y′ = x

Let Q =(y′ , x) andthus Q willbeanewlyconstructedpoint inthefirstquadrantonthehigherbranchof H,andwith smaller x,and y-coordinatesthanthepoint P westartedwith. Ifwerepeatthisprocess,the x-coordinatewillformasequenceofdecreasingnon-negativeintegers.Thiscanonly berepeateduntil Q =(0, y′) as x ∈ Z Bysubstitution, q = y2 .

7Conclusion

WhenVietaJumpingwasfirstintroducedin1988,itwas unheardofandthusitbecameveryfamousveryquickly. However,asmorethan30yearshavepassed,thistechnique isnowstandardisedandthusitdoesnotseemtoappearin Olympiadsoften.ThisisbecausetheproblemsthatuseVieta Jumpinghavesolutionsthatarebasicallythesame(asseen inthe3examplesabove)andalsoitisquiteeasytorecognise.Furthermore,althoughitiscalledVietaJumping,inthe end,itusesalargeaspectofFermat’smethodofinfinitedescent,whileFermat’smethodofinfinitedescentdependson theWell-orderingprinciplewhichisoneofthemostimportanttheoriesinnumbertheory.Yet,thistechniquewasonce knownasoneofthemostbeautifulwaystoproveaquadratic Diophantineequation.

8Bibliography

1.Brilliant.org.(2025).VietaRootJumping—BrilliantMathScienceWiki.[online]Availableat: https://brilliant.org/wiki/vieta-root-jumping/[Accessed 23Jan.2025].

2.Tat-Wing,L.(2005).TheMethodofInfiniteDescent.[online]Availableat: https://www.math.hkust.edu.hk/excalibur/v10n4.pdf [Accessed19Oct.2024].

Figure1.SketchofhyperbolaH.

TheTaylorSeries

Year10

Email:jyseol28@pupils.nlcsjeju.kr

RecommendedYearLevel:KS4 Keywords:Taylorseries, MaclaurinSeries

1Introduction

Thetaylorseriesisafundamentalmethodtoapproach afunctionincalculus.Thisconceptrepresentsfunctionsas aninfinitesum,andmodelstheclosestapproximationtothe originalfunction.

2TheTaylorSeries

Thetaylorseriesisakeyconceptincalculus,andisa representationofacontinuous,differentiablefunctioninto aninfinitesumofpolynomials.

Thisisaninfinitesumoftermsthatareexpressedin termsofthefunction’sderivativesatasinglepoint.

2.1TheMaclaurinSeries

TheMaclaurinseriesisasubsetofthetaylorseries wherethefunction’sderivativeisconsideredissetat0.Takingthegraph cos(x) asanexample.Letafunctionbe P(x), where P(x)= c0 + c1x + c2x2,andourgoalistomakethis polynomialaclosestapproximationto cos(x).

Tomakethispolynomialsimilartothegraph cos(x) as possible, c0,the y-interceptmustequatetothe y-interceptof thegraph cos(x).Therefore, c0 = 1.Thegradientof P(x) shouldalsoequatetothelocaltangentslope.Thederivative of cos(x) equalsto sin(x),and x = 0.Therefore,thelocal tangentslopeofthefunctionis0.Takingthederivativeof P(x),

d dx (x0 + x1x + c2x2)= 0

c1 + c2x = 0

x = 0

c1 + c2(0)= 0

c1 = 0 P(x)= 1 +(0)x + c2x2

Nowsolvingfor c2,Thesecondderivativeof cos(x) equatesto cos(x). x = 0,thereforeitis 1.Takingthe secondderivativeof P(x):

d2 dx2 (c0 + c1x + c2x2)= 1

2c2 = 1

c2 = 1 2

P(x)= 1 +(0)x +( 1 2 )x2

Graphing P(x))alongwith cos(x) showsanapproximation

near x = 0:

Figure1.P(x)andcos(x)sketched

LetthepolynomialP(x)havemoretermswithlarger exponents:

P(x)= 1 +(0)x +( 1 2 )x2 + c3x3 + c4x4

Applyingthethirdandthefourthderivative,

d3 dx3 cos x = sin x sin0 = 0

d3 dx3 (1 +(0)x +( 1 2 )x2 + c3x3 + c4x4)= 0 0 + 1 × 2 × 3 × c3 = 0

c3 = 0

d4 dx4 cos x = cos x cos0 = 1 d4 dx4 (1 +(0)x +( 1 2 )x2 +(0)x3 + c4x4)= 1

0 + 1 × 2 × 3 × 4 × c4 = 1 c4 = 1 24

P(x)= 1 +(0)x +( 1 2 )x2 +(0)x3 + 1 24 x4

Figure2.developedversionofP(x)(blue)andcos(x)(red)sketched

Thegraphaboveillustratesamuchaccurateestimation tothegraph cos(x) near x = 0.Here,onemustrecognise thatcoefficient cn equalstothederivativevalueover n!.For instance,the4thderivativeoftheterm c4x4 equalledto1 × 2 × 3 × 4 × c4,whichequalsto4! × c4 .Moreover,onemust notethataddingnewtermsdonotinfluencethevaluesofthe otherterms.Thisisbecausesubstituting x as0alwaysoccur inthisprocess.Forinstance,thesecondderivativeof P(x) doesnotinterferewiththeotherterms,becausetheprevious termsallbecome0andthetermsafteritcancelsoutas x becomes0,shownbelow.

0 +( 1 2 )+ c3(0)1 + c4(0)2

Thecoefficientsofthetermsarethenthderivativesofof thefunction P(x) when x is0.Therefore,foreachterm,a generalisedexpressioncouldbe:

Forconvenience,letusdefine fn(0) asthenthderivative of f at0,thereforetheexpressionbeing:

( f n(0) n! )xn

Giventhatthefunction P(x) getsclosertotheinitial function cos(x) whenmoretermsareadded,itispossible toacquireinfiniteamountoftermsinorderforthefunction P(x) tobethemostsimilarwiththefunction cos(x)

Therefore,usingthegeneralisedexpression,aninfinite sumofthetermsarerepresentedas:

3TheEulerFormula

Thefamouseulerformulawhere eix = cosx + isinx is oftenrecognisedasthemostbeautifulformulainmaths because:

eiπ + 1 = 0when x = π Thisformulaisprovenfromthe taylorseriesitself.Letusfirstconsiderthederivativesof eix toobtainthetaylorseriesof ex ex isaveryconvenient exampleasthederivativeof ex equalstoitself.Therefore, onecanassumethatthecoefficientsofthetermsareall1. Hence,thetaylorseriesof ex is:

Replacing x with ix gives:

Since i is √ 1, i2 resultsin 1.Henceforth,thetaylor seriesfor eix becomes:

2.2TheTaylorSeries...Again

Thetaylorseriesisamoregeneralisedformulawhere thederivationpointisabletochange.

Let a bearealorcomplexnumbersothat x a isthe derivationpoint.

Therefore,byreplacing0with a,since f n isthe nth derivativeof f evaluatedatacertainpoint,andbychanging x to x a inthemaclaurinseries:

Thisexpression,whichisthetaylorseriesisobtained.

Groupingtheimaginarypartgives:

Calculatingthederivativesof cosx andusingthemaclaurinseriestoset x = 0: d dx cos x = sin x = 0 d2 dx2 cos x = d dx ( sin x)= cos x = 1 d3 dx3 cos x = d dx ( cos x)= sin x = 0 d4 dx4 cos x = d dx (sin x)= cos x = 1

Anditrepeats.

Therefore,onecanacknowledgethatthecoefficientforthe termsareinfinitecyclesin0, 1, 0, and1.

Knowingthis,thetaylorseriesforcosxis: cos(0)+(0) x 1! +( 1) x2 2! +(0) x3 3! +(1) x4 4! +

Whichsimplifiesto: 1 x2 2! + x4 4! +

Lookingbacktothetaylorsereisof eix,whichwas: 1 x2 2! + x4 4! + + i(x x3 3! + x5 5! + )

showsthatthetaylorseriesfor cosx equalstoacertainpartof theTaylorseriesfor eix.Replacingthefirstsequencegives: eix = cos x + i(x x3 3! + x5 5! + )

Calculatingthederivativesof sinx andusingthe Maclaurinseriestoset x = 0:

Anditrepeats.

Therefore,onecanacknowledgethatthecoefficientforthe termsareinfinitecyclesin1, 0, 1, and0.

Knowingthis,thetaylorseriesforsin x is:

+(1)

Whichsimplifiesto:

Lookingbacktothetaylorseriesof eix:

Showsthatacertainpartofthetaylorsseriesequatestothe taylorseriesofsin x

Replacinggives:

4Limitations

TheTaylorseriesisapowerfulmathematicaltoolused toapproximatefunctionsusinganinfinitesumofterms derivedfromthefunction’sderivativesatasinglepoint. However,ithaslimitations.

Onekeylimitationisitsrelianceonthefunctionbeing infinitelydifferentiableatthepointofexpansion.

Pointingouttheobvious,Ifthefunctionisnotsufficientlysmooth,suchashavingdiscontinuities(since differentiationonlyappliesoncontinuousfunctions,the taylorserieswouldobviouslynotwork)orundefinedhigherorderderivatives,theTaylorseriesmayfailtoconvergebut diverge.

Anotherlimitationarisesfromtheconvergenceofthe series.Evenforsmoothfunctions,theTaylorseriesmaynot convergetotheactualfunctioneverywhere.

Forinstance,theseriesexpansionfor f (x)= e 1 x2 /x2 at x = 0yieldsaTaylorseriesequalto0forall x asany derivativeofthisfunctionis0.Thisisbecause e 1 x2 dominates thecoefficients,tendingto0faster.

Thisdiffersfromthefunction’struevalueof f (x) > 0 for x = 0.

pointbetween a and x

Thismeasureshowfaristhetaylorpolynomialfromthe actualfunctionatthenextterm, n + 1,andisoftenreferred toastheLagrangeErrorBound.

If Rn(x) doesnotapproach0as n −→ ∞,theseriesfails toapproximatethefunctionaccurately.

5Bibliography

1.MathisFun.(2017).TaylorSeries.Mathsisfun.com. https://www.mathsisfun.com/algebra/taylor-series.html

2.Dawkins,P.(2020,May26).CalculusII -TaylorSeries.Tutorial.math.lamar.edu. https://tutorial.math.lamar.edu/classes/calcii/TaylorSeries.aspx

3.Taylorseries—Chapter11,Essenceof calculus.(n.d.).Www.youtube.com. https://www.youtube.com/watch?v=3d6DsjIBzJ4abchannel=3Blue1Brown

4.Ault,Shaun.“APCalculusBCReview:Lagrange ErrorBound.”MagooshBlog—HighSchool,10 Aug.2017,magoosh.com/hs/ap/ap-calculus-bc-reviewlagrange-error-bound/.

5.“TaylorSeries-ErrorBounds—BrilliantMath ScienceWiki.”Brilliant.org,brilliant.org/wiki/taylorseries-error-bounds/.

If f (x) isexpandedabout x = a,theerrorinthen-thdegreeTaylorpolynomial Tn(x) isexpressedastheremainder: Rn(x)= f (x) Tn(x)= f n+1(c) (n+1)! (x

Where c issome

RecommendedYearLevel:KS3,KS4

Keywords:irrational,graph,inequality

JunyoungKim Year10

Email:jy2kim27@pupils.nlcsjeju.kr

1Introduction

Theconstants π and e arefundamentalinmathematics,appearingacrossvariousareassuchasgeometry, calculus,andnumbertheory.Oneintriguingquestion thatarisesisthecomparisonoftheirexponentialforms: πe and eπ

Thisarticleprovidesaseriesofproofsandinsights intotherelationshipbetweenthesetwoexpressions, sheddinglightontheirfascinatingandunexpectedbehaviour.

2BackgroundInformation

Thenumberpi,writtenas π istheratioofthecircumferenceofanycircletoitsdiameter.Itisafundamentalconstantinmathematics,appearinginvarious formulasingeometry,trigonometry,andcalculus.More formally, π isdefinedas:

3Method

Therearethreemainproofspresented.Thefirst twoproofsinvolvegraphicalanalysis,whilethethird utilizestheMeanValueTheorem(MVT)toprovidean alternativeperspectiveontheinequality.

3.1Proofusingthegraph y = xe e x Let’sbeginbydefiningf(x)asfollows:

y = xe · e x

Thegraphof y = f (x) increaseswhen 0 <x<e and decreaseswhen x>e

Fig.1: y = xe e x

Thisdefinitioncomesfromthelimitofaninscribed polygonwithnsides,buthasnumerousotherrepresentationsandappearancesinmathematics.

Thenumber e isthebaseofthenaturallogarithm. Itisdefinedastheuniquerealnumbersuchthatthe derivativeofthefunction f (x)= ex isthesameas ex itself,makingitthebase ofexponentialgrowth.Moreformally, e canbedefined bythefollowinglimit:

Fromthegraph, f (e)=1,π e e 1 < 1

whichcanberearrangedinto

π e <eπ

Therefore, πe <eTπ

3.2Proofusingthegraph y = 1 x Thegraph y = xe e x mightbeabithardto sketch,soanotherproofusingagraphisprepared.

Let’sdefine f (x) asfollows: f (x)= 1 x

Fig.2: y = 1 x

Fromthegraphabove,wecanknowthat

Since

)

And 1 e (π e)= π e 1

Wecansubstituteandrearrangetheinequalityto get

ln π< π e e ln π<π

ln π e <π π e <eπ (1)

4Proofusingmeanvaluetheorem Themeanvaluetheoremstatesthatwhenafunction f (x) iscontinuousontheclosedinterval [a,b] and differentiableontheopeninterval (a,b) where a<b, thereexistsatleastone c in (a,b) suchthat:

f ′(c)= f (b) f (a) b a

For f (x)=ln x,themeanvaluetheoremtellsusthat forsome c ∈ (e,π),

f ′(c)= f (π) f (e) π e

Whichbecomes 1 c = ln π ln e π e = ln π 1 π e

Since e<c<π 1 c < 1 e (2)

Sowehave:

ln π 1 π e < 1 e

ln π 1 < π e e

ln π< 1+ π e e

ln π< π e e ln π<π ln π e <π π e <eπ (3)

5Conclusion

Inthisarticle,weexploredthefascinatinginequalitythroughthreedistinctapproaches:graphicalanalysesandaproofleveragingtheMeanValueTheorem. Eachmethodoffersauniqueperspectiveontherelationshipbetweenthesetwoexponentialexpressions,ultimatelyleadingtothesameconclusion.

Theinequality πe <eπ servesasaremarkableexampleofhowseeminglyabstractconstantslike e and π,

whichoriginateinentirelydifferentareasofmathematics,interactinwaysthatarebothsurprisingandbeautiful.Theseresultsnotonlydeepenourappreciation fortheinterconnectednessofmathematicalconceptsbut alsohighlightthepowerofdiversemathematicaltools andtechniquesinuncoveringsuchrelationships.

6Bibliography

1. Artelius,J.2010."AQuestion Comparing π to e]pi."Mathematics StackExchange.October20,2010. <https://math.stackexchange.com/questions/337565/aquestion-comparing-pie-to-e-pi>

2. Rayc20.2020."WhichisGreater,etothePower of π or π tothePowerofe?"Tistory.February17, 2020<https://rayc20.tistory.com/14>

ExploringtheRelationshipBetweenConstraints andFeasibleRegionGeometryinLinear Programming

Email:jwyu27@pupils.nlcsjeju.kr

RecommendedYearLevel:KS4

Keywords:LinearProgramming,FeasibleRegion, Convexity

1Abstract

Linearprogrammingisamathematicalmethod usedtooptimizeanobjectivefunction,subjecttoaset ofconstraints.Itiswidelyappliedinareaslikeeconomics,engineering,andlogistics.Thefeasibleregion, definedasthesetofallpossiblesolutionsthatsatisfyall givenconstraints,playsacriticalroleintheoptimizationprocess.Thegeometryofthisregionisinfluenced bytheconstraints,whicharetypicallyrepresentedby linearequationsorinequalities.Akeypropertyofthe feasibleregionisconvexity,meaningthattheregionis well-structured,andoptimalsolutionsareguaranteedto lieatitsvertices.Thisjournalexaminestherelationship betweentheconstraintsandthegeometryofthefeasibleregion,focusingonhowtheconstraintsinfluence itsshape,structure,andconvexity.Byunderstanding thesegeometricproperties,thejournalprovidesinsights intohowconstraintsimpacttheoptimizationprocessin linearprogramming.

2Hypothesis

Thehypothesisisthattheconstraintsnotonlydeterminethesizeandshapeofthefeasibleregionbutalso controlitsconvexityandtheplacementofitsvertices. Asthenumberofconstraintsincreases,thefeasibleregionisexpectedtoshrinkorbecomemorecomplex,but itsconvexitywillbepreserved.Additionally,thepositioningandinteractionoftheconstraintswillinfluence thegeometryoftheregioninwaysthatarecrucialfor

identifyingoptimalsolutionsinlinearprogramming.

3Methodology

1. SetupofTestProblems: Constructabaseproblemintwodimensions,startingwithnon-negativity constraints:

x1 ≥ 0,x2 ≥ 0

Incrementallyintroduceadditionallinearconstraints(forexample, x1 + x2 ≤ 6 or 2x1 + x2 ≤ 8) toobservechangesinthefeasibleregion.

2. GeometricAnalysis: Ploteachconstraintasa line(in2D)oraplane(in3D)tovisualizehow thefeasibleregionisalteredwhennewconstraints areadded.Identifyverticesforeachversionofthe problemtoseewhetheroptimalsolutions,whenan objectiveisapplied,lieatthosecornerpoints.

3. ConvexityVerification: Checkthatforany twopointsinsidethefeasibleregion,thelinesegmentconnectingthemstaysentirelywithinthatregion.Examinewhethertheadditionofconstraints changestheoverallshapewhilepreservingconvexity.

4. ObservationofRedundancyandContradiction: Identifyconstraintsthatdonotfurtherlimit thesolutionspace(redundant)orconstraintsthat leadtonofeasiblesolutions(contradictory).Note howredundantconstraintsdonotchangethegeometry,whilecontradictoryconstraintscaneliminate thefeasibleregionaltogether.

Toinvestigatehowconstraintsinfluencethegeometryofthefeasibleregion,asystematicapproachwas used,beginningwithaminimalsetofconstraintsand graduallyaddingormodifyingthem:

BaseCase:MinimalConstraints Startingwith x1 ≥ 0 and x2 ≥ 0 confinestheregiontothefirst quadrantofthe x1- x2 coordinateplane.Thisunboundedregion(thefirstquadrant)isconvex:any twopointsinthefirstquadranthaveaconnecting linesegmentthatremainsinthatquadrant.

AddingaSingleLinearConstraint Theintroductionof x1 + x2 ≤ 6 restrictsthefeasibleregionto atriangularshapewithverticesat(0,0),(6,0),and (0,6).Thisshapeisstillconvex,andthevertices areclear.

Fig.1:Thetriangularshapeofthefeasibleregionshown above

IncrementalConstraints Additionalconstraints suchas 2x1 + x2 ≤ 8 or x1 ≤ 4 furtherrefinethe feasibleregion,“cuttingoff”partsoftheexisting region.Theseconstraintsmaytransformthetriangleintoapentagonoranotherpolygonalshape,but theregionremainsconvexunlesstheconstraintsare contradictory.

TestingConvexityToverifythatagivenfeasible regionisconvex,astandardapproachistocheck whether,foranytwopointswithintheregion,everypointonthelinesegmentconnectingthemremainsinsidetheregion.Whileconvexitycanbevisuallyinterpretedbyobservingtheshapeofthefeasibleregion,analgebraicapproachprovidesamore preciseandreliablemethod,especiallyforanalyzingtheconvexityofcomplexshapes.Let (x1,x2) and (x′ 1,x′ 2) betwofeasiblepoints,andtheexpressionbelowrepresentsthelinesegment/pointsbetweenthetwofeasiblepointswhere λ ∈ [0, 1] f (λ)= (λx1 +(1 λ)x′ 1), (λx2 +(1 λ)x′ 2) If f (λ) conformstoallconstraintsforevery λ ∈ [0, 1],thentheregionisconvexForourcase,the constraintswere x1 ≥ 0, x2 ≥ 0, x1 + x2 ≤ 6 andthe verticesofthetriangularregionare(0,0)(6,0)and (0,6).Points(2,2)and(4,1)werechosenarbitrarily.Thelinesegmentconnectingthesepointswas thenevaluatedtodeterminewhetherallintermediatepointssatisfythegivenconstraints.

(λ)=(λ(2)+(1 λ)(4),λ(2)+(1 λ)(1))

Expanding:

Overall: x1 =4 2λ ≥ 0 (alwaystruefor λ ∈ [0, 1])

2 =2λ +1 ≥ 0 (alwaystruefor λ ∈ [0, 1]) (4 2λ)+(2λ +1)=5 ≤ 6 (alwaystrue) Thustheregionisconvex.

4Conclusion

Thisjournalexploredtherelationshipsbetweenconstraintsandthegeometryofthefeasibleregioninlinearprogramming,withaparticularfocusonconvexity. Throughmathematicalanalysis,itwasdemonstrated thatincreasingthenumberofconstraintsaltersthesize andshapeofthefeasibleregionwhilepreservingitsconvexity,providedtheconstraintsremainconsistent.The convexitytestconfirmedthatanylinesegmentbetween twofeasiblepointsremainswithintheregion,reinforcingthefundamentalpropertyoflinearprogramming thatoptimalsolutionslieatvertices.Understanding thesegeometricpropertiesisessentialforoptimization, asitensuresthatfeasibleregionsmaintainawell-defined structure,allowingforefficientsolution-findingmethods suchastheSimplexalgorithm.

GoldenRatioinHigherDimensions

Email:mjkang28@pupils.nlcsjeju.kr

RecommendedYearLevel:KS4,KS5

Keywords:GoldenRatio,analysis,expansion

Editor TaehongAustinHa

1IntroductiontotheGoldenRatio

Thegoldenratio,denotedbytheGreekletter ϕ is amathematicalconstantthathasintriguedmathematicians,artists,andscientistsforcenturies.Algebraically, itcanbesolvedby a + b a = a b (a>b> 0)

Whichleadsto x 2 = x +1

Whichsimplifiesto x 2 x 1=0

Usingthequadraticformulatosolveforx,wefind thetwosolutions

x = ( 1) ± ( 1)2 4(1)( 1) 2

Thepositiverootofthisequation, 1+ 5 2 isthe goldenratio.Indecimals,thisvalueisapproximately 1.618.

Thisvaluerepresentstheratiooftwoquantitiesthat areaestheticallypleasing,appearingfrequentlyinmany artformsandevennature.

Whilemostdiscussionsofthegoldenratiofocuson twoandthreedimensionalobjects,ithasfurtherimplicationsinhigher-dimensionalspacesaswell.Inthese spaces,thegoldenratioisusedtodescribeproportions thatmaximisesymmetryandbalance,contributingto structuralstability.

Thearticlewillexploreonhowthegoldenratiocan beappliedtohigher-dimensionalspacesusingtheidea of“goldenboxes”,whicharemultidimensionalobjects whereeachdimensionisscaledaccordingto ϕ.The focuswillbeondeterminingthestabilityoftheboxes byexaminingthesurfaceareatovolumeratiosbasedon thefactthatstabilityisachievedwhenthesurfacearea tovolumeratioisminimised.

2N-dimensionalgoldenbox

Letthen-dimensionalgoldenboxhavesidelengths thatfollowthegoldenratio

Ifweletthesidelengthofthefirstdimension, x1 ,tobea,thesidelengthofdimensionwillbegivenby

xk = a ϕk 1 for k =1, 2....n

3Volumeofthen-dimensionalgoldenbox

Thevolume,V,ofthen-dimensionalgoldenboxis theproductofitssidelengthsofdifferentdimensions

V = x1 x2 xn

Substitutingeach xk with a ϕk 1

V =(a ϕ) (a ϕ2).... (a ϕn 1)

Usingtheformulaforthesumof n 1 integers, 0+ 1+2+ .... +(n 1)= n(n 1) 2 ,thissimplifiesto an ϕ n(n 1) 2

4Surfaceareaofthen-dimensionalgoldenbox

Thesurfaceareaofann-dimensionalboxrefersto thesumofallareasofthen-1dimensionalfaces.

Foreachdimensionk,therearetwofaceswitharea givenbytheproductofallsidelengthsexcept xk Summingtheareasofallthesefacesweget:

SurfaceArea=

5Surfaceareatovolumeratio

Therefore,ratioRofthesurfaceareatovolumeof n-dimensionalgoldenboxisasfollows:

7Applicationofn-DimensionalGoldenBoxes

Duetoitsrelativestability,thegoldennboxin higherdimensionscanbeusefulinvariousfields.Two ofthefieldsarearchitectureandnaturalsciences.

7.1Architecture

Inarchitecture,theideaofn-dimensionalgolden boxescanaffectthedesignofcomplexstructures.The constantproportionalityinthesegoldenboxesenables evenstressdistributionacrossstructures,ensuringbalanceanddurability.Furthermore,stability,orminimizingthesurfaceareatovolumeratio,reducestheamount ofmaterialrequired,presentingapossibleopportunity tolessenthecostburdenforarchitects.

7.2NaturalSciences

Thegoldenratiohaslongbeenassociatedwith naturalgrowthpatterns.Theintroductionofndimensionalspacescanprovidedeeperinsightintohow creaturesmaintainstabilityandefficiencyinsurvival. Specifictopicsforresearchincludecellulargrowth, moleculararrangements,oreventheorganizationof ecosystemsinhigher-dimensionalspaces.

8Conclusion

Thegoldenratio,widelyknownforitsaesthetic qualities,continuestorevealitssignificanceinmultidimensionalspaces.Thiscanbeshownbytheinvestigationofn-dimensionalgoldenboxes,whichrevealsthat thesurfaceareatovolumeratioisconstant,andthus isstableinhigherdimensionsaswell.Thismakesita usefultoolfordesigningsystemsthatrequirestructural integrity.

Thisindicatesthat,foraconstantinitialdimension lengthofa,thesurfaceareatovolumeratioisrelativelystableasdimensionsincrease,andthusthegolden n-boxesproportionallymaintainefficiencyinhigherdimensions.

6Stability

Stabilityisachievedwhensurfaceareatovolume ratioisminimised.Extendingthisconcepttohigherdimensionalspaces,theconstancyof R ingoldenboxes indicatesstability.Therefore,itcanbeusefulinstructuresinmultipledimensionsthatrequireuniformityand balance.

Euler’sProofofTheInfinitudeofPrimeNumbers

SeongminHong Year12

Email:smhong26@pupils.nlcsjeju.kr

RecommendedYearLevel:KS4,KS5

Keywords:Primenumbers,Riemannzetafunction, Numbertheory

1Introduction

Iamcertainthatmostpeoplereadingthisarticle willhaveencounteredEuclid’sProofoftheInfinitudeof PrimeNumbers,beingaclassicexampleofa reductio adabsurdum.Assumingotherwise,andthatafinite numberofprimesexist,Euclidproposedthattheremust existaset P ofprimesfrom p1 upto pn (where p1 = 2,p2 =3,...).Thatmeansthat

= n i=1

+1=(

mustbeindivisiblebyanyoftheprimeswithinthe setofprimesthatwe’veestablished,meaningthere hastoexistanewprimenumber,meaningthrough contradiction,therecannotexistafinitesetofprimes. Hence,Euclid’sargumentguaranteesthata’largest’ primenumbercanneverexist.

Forexample,ifweweretoassumethatthelargest primenumbertoexistwas11,thentheproductof ourprimes q =(2 × 3 × 5 × 7 × 11)+1=2311 wouldbe indivisibleby 2, 3, 5, 7 and 11.Andhenceitstandsto reasonthat 2311 isaprimenumber(whichitinfact is!),orthereexistsanotherprime pn+1 < 2311 which isnotinoursetofallprimenumbers.Eitherway,a contradictionoccurswithourinitialassumption,and hence 11 isnotthebiggestprimenumbertoexist,and norisanyotherprimenumberbythesamelogicalflow.

Whilsttheeleganceofthisproofiscleartosee,it disappointswhenattemptingtoestimatingthedensity ofprimes.Thisargumentstatesthatthenextprime

numberafter p1 to pn couldbeasbigas q = n i=1 pi +1

Fortunately,Eulerstrengthensthedensityofprimes throughcomingupwithadistinctprooffortheinfinitudeofprimenumbers,usingwhatwouldlaterbe knownastheRiemannzetafunction,andevenpaves thepathforthePrimeNumberTheorem.

2Euler’sProof

Eulerarguedthat

where P isthesetofallprimes,and a ∼ b as s → 1+ means a and b areasymptoticallyequivalent,or a b approaches 1 as s tendsto 1+ fromabove.

TheRiemannzetafunction,alsoknownasthe Euler-Riemannzetafunction,isafunctionofacomplex variable,andhasmanyuseswithinanalyticnumbertheory(themostfamousofwhichistheRiemannhypothesis,whichisaboutthezeroesoftheRiemannzetafunction).Itisdefined(ignoringtheintegralform,complex natureandtheanalyticalcontinuationforRe(s) < 1)as being

Wewanttolinkthezetafunctiontothefraction 1 s 1 ,sincethisfractionappearsin(1),andthiscanbe foundinthebyanalysingtheareaunderthecurveof y = 1 xs throughintegration.

Fig.1:Themodularcurve

Fromthegraphabovewecanseethatthearea underthecurveof x s isclearlylinkedtotheRiemann zetafunction.Thesumoftheareasofthebluelesser shadedrectangle ( 1 1s + 1 2s + 1 3s + 1 4s + ) isostensibly equaltotheRiemannzetafunction,butalsogreater thantheintegralofthecurve ∞ 1 x sds,sincethese boxesareanoverestimation.Thesumoftheareasofthe blueheavilyshadedrectangle ( 1 2s + 1 3s + 1 4s + 1 5s + ) is similarlyequaltotheRiemannzetafunctionminus 1, butalsolesserthantheintegralofthecurve ∞ 1 x sds, sincetheseboxesareanunderestimation.

Thatmeansthataninequalitymaybeformedbetweenfromthis:

Hence,takingthenaturallogarithmofbothsides,

Itmustbenoticedthattherighthandsideof(1)is equaltotherighthandsideof(3).Hence,itissatisfactorytoprovethat

Thegivenboundsimplythat

Consequently,bythesqueezetheorem,thisshows that

Subtractingtheaboveequationfrom(2),weget

Noticethatthedenominatoroftheaboveequation containnomultiplesof2.Thatmeansthat

Withthesameprocess,subtractingtheaboveequationfromtheequationpriortoit,weget

Similarly,thedenominatoroftheabovetheequationnowcontainnomultiplesof2or3.Whathappens whenwerepeatthisprocessinfinitelybymultiplying bythereciprocalofeveryprimetothepowerofs,ina mannersimilartothesieveofEratosthenes?

Dividingbothsidestoonlygetthezetafunctionon thelefthandside,

Therefore,takingthenaturallogarithmofboth sides

ln(ζ(s))=ln

SincetheMaclaurinexpansionofthenaturallogarithmintheformfoundaboveis

ln(1 x)= x

TheMaclaurinexpansionaboveisonlyapplicableifthe itpassestheratiotest.Hence,

SubstitutingtheMaclaurinexpansionof ln(1 x),

ln(ζ(s))=

Sincewearetryingtoprove(4),weneedtoshow thatthefinalpartoftheaboveequationisextremely smallcomparedtotheothertwotermsintheequation as s tendsto 1+.First,notethat:

As s tendsto 1+,thevalueofthelefthandside willalwaysbelessthan 1,butbothindividualterms willgrowto ∞.Thisproves(4),whichinturnwas aconditionsatisfactorytoprovetheinfinitudeofthe primenumbers.

Then,wemayfindthattheinnersumcanbeexpressedasageometricseries.

As s → 1+ , ln 1 s 1 willtendto ∞,andsince p∈P 1 ps isasymptoticallyequivalent,itwillalsotend to ∞.Iftherewereafinitenumberofprimes,thesum foundabovewouldconverge,sinceafiniteseriescannot diverge.Hence,thereareaninfinitenumberofprimes.

3ThePrimeNumberTheorem

Inthepaperthatcontainedtheaboveproof,called Variaeobservationescircaseriesinfinitas,or"Various observationsaboutinfiniteseries",Eulerstates"Atque illiussummaesthuiussummaequalilogarithmus""Andthesumofthat(thereciprocalsoftheprimes) isthelogarithmofthissum(ofthereciprocalsofthe naturalnumbers)".Thisismathematicallysuggesting that

Hence,

Noticethefirstinequalityholdsasaresultoftheset ofprimenumbersbeingasubsetofthesetofnatural numbers,andthesecondinequalityholdssince ns(ns 1) >n(n 1) ⇒ 1 ns(ns 1) < 1

.Uponexpansion,

However,hisprooffoundwithinthebookwasnot rigorous,anditisnotknownwhetherhecouldproveit rigorously.

Letusanalysewhatthisobservationleadsto though.Toestimatethedensityoftheprimes,wecan differentiatebothsideswithrespecttonusingthechain rule.

dn ln(ln n)= d ln(ln n) d ln n d ln n dn = 1 n ln(n) 1 n = 1 n ln(n)

Thisshouldshowushowmuchthesumisexpected toincreasewhenincrementingfromanygiven n to n+1 Considerthelefthandsideof(6).If n wereprime, thenthelefthandwouldbeexpectedtoincreaseby 1 n

andifotherwise,itwouldbeexpectedtoincreaseby 0 Sincethederivativeoftherighthandsideis 1 n ln(n) ,the probabilityof n beingprimemustbe 1 ln(n) .Thismeans that

π (x)= p∈P,p<n 1 ∼ x ln(x))

⇐⇒ π (x) x ∼ 1 ln(x)) forlargervaluesof x (7)

andsince π(x) representsthenumberofprimesless thanorequalto x, π(x) x mustrepresenttheprobability of x beingaprimenumber,whichgivesasacorollary thePrimeNumberTheorem.

Thediscoveryoftheprimenumbertheoremcould becreditedtoLegendre,whopositedthetheoremin 1798,orGaussinhisnotebookpriorto1800,andwas laterprovenin1896usingsomerigorousmathematics. However,itwouldn’tbeastretchtosaythattheprime numbertheoremwouldhavebeenaneasycorollaryby Eulerifhehadtrulyobservedandproven(6).

4EndingRemarks

WhetherEulerdidreallyobserve(6)willneverbe certain,buttheproofof(1) (=(5)) wasstilloneof thebiggestadvancementsmadeinthefieldofprime numberssinceEuclid’sproofoftheinfinitudeofprime numbersovertwomilleniaprior,inspiringmanytocontinuedownhispathintoprimenumbersandanalytical numbertheory.Notonlyisthisproofthefirstever appearanceoftheEuler-ProductformulafortheRiemannzetafunction,butitwasthetoolwhichDirichlet modifiedtoproveanevenstrongerresultaboutprime numbers,knownasDirichlet’stheorem.

5Bibliography

1. Conrad,K.(2023).Theinfinitudeofthe primes.[online]kconrad.math.uconn.eduAvailable at:https://kconrad.math.uconn.edu/blurbs/ugrad numthy/infinitudeofprimes.pdf

2. WolframMathWorld.(2018)RiemannZetaFunction.[online]mathworld.wolfram.com.Availableat: https://mathworld.wolfram.com/RiemannZetaF unction.html

3. ThinkingInMath(2022).Euler’s Proof-ThereAreInfiniteManyPrimes [video]youtube.com.Availableat: https://www.youtube.com/watch?v=DEmoPfeAA KA

Fermat’sLittleTheorem

Taewon(Ryan)Kim Year10

Email:twkim28@pupils.nlcsjeju.kr

RecommendedYearLevel:KS4

Keywords:ModularArithmetic,PrimeNumbers, CompositeNumbers,PrimalityTesting,Carmichael Numbers,BinomialTheorem

1Introduction

In1679,oneFrenchmathematicianintroducedan innovativeconceptwhichlateronwasincorporatedin provingnumeroustheories;thisisFermat’slittletheorem.Thetheoremstipulatesthatif p isaprimenumber, thenforanyintegera,

Theequationaboveindicatesthattheremainderof ap dividedby p iscongruent—equal—totheremainder ofadividedby p.Anotherequationdrivenbythefirst onewiththeconditionthataiscoprimeto p is,

p 1 ≡ 1(modp)

Whentheequationaboveissatisfied,aslongasa and p arecoprime,theremainderof a(p 1) divided bypalwaysresultsas1.Hence,thetheoremcanbe implementedtoderivewhetheracertainintegerisa compositenumberbydeterminingifthecertaininteger satisfiestherelationornot,ortoderivetheremainder ofanumberforparticularcases.

2BackgroundInformation

PierredeFermat,arenownedmathematicianinthe 1600s,introducednumerousproofsandvaluablemathematicalinsightstotheworldbackthen,whichgreatly

contributedtothedevelopmentoftheparticularfield. Throughouthiswell-respectedlife,Fermathadrather apersonalmethodofshowcasinghisnewfoundknowledge.Insteadofovertlypublishinghisproofsandtheoremstothepublic,histheoremswerepresentedmainly throughlettersormessagesinaprivatemanner,devoid ofmathematicalproofs.Famously,Fermatpresenteda theorythatwasdubbedFermat’sLastTheoreminhis book,which,however,onlyencompassedtheprinciples ofthetheoremwithoutproofs,sayingthattherewasn’t enoughspacetowriteitdown:"Ihavediscovereda trulymarvellousproofofthispropositionthatthismarginistoonarrowtocontain(Fermat’sLittleTheorem wasalsofirstintroducedinalettersenttohisfriend). Duetosuchabluntmannerofintroducingstatements andwiththeabsenceofproof,alotofmathematicians duringhistimeandinlatercenturiesquestionedhis work.However,asfuturemathematicianssucceededin corroboratingFermat’sstatementsonebyone,hisabsolutemathematicalingenuitywaswidelyrecognisedin thepresentday.

3Proof

3.1CombinatorialProof(ProofbyCounting Necklaces)

ThefollowingproofisanadaptationofGolomb’s proofofFermat’sLittleTheorem,whichcanbemost explicitlydisplayed.

Firstofall,imagineanecklace,andletthenumber p,apositiveinteger,bethenumberofbeadsofthe necklace.Theaiminthiscontextistofullycolourthe beadsofthenecklaceonebyone.Letanotherpositive integer,a,bethenumberofcoloursavailable.Ifall possiblecombinationsofstringswhereeachstringis constructedofpnumberofalphabetswithdifferent possiblealphabetsweretobecalculated,thetotal numberofthosestringswouldbeapaccordingtothe ruleofproduct.

Forexampleifp=5anda=2,therewouldbe25 =32strings.

Fig.1:32Strings

However,thestringsthatconsistofasinglesymbolwouldberemovedfromthislist,astheabovehas restrictedthedifferentpossiblealphabetsto2.Anyelementthatunsatisfiestheconditionistoberemovedin thisproof.Inthecaseabove,AAAAAandBBBBB, whichare2strings—a numberofstrings—wouldbe deleted.Henceforth,theremainingnumberofstrings wouldresultasap-a,wheretherestofthestringscan bearrangedintogroupscontainingpnumberofstrings.

Fig.2:30Strings

Inthecircumstanceabove,therefinedlistcontains 25 2=30 stringswhereeachgroup,column,isconstructedof5strings.Therefore,itcouldbestatedthat it ap a isdivisibleby p,inthiscase 30/5=6,corroboratingtheequationwhichargues ap ≡ a(modp).

3.2ProofusingtheBinomialTheorem

AnotherformofproofforFermat’sLittleTheorem incorporatestheconceptsofthebinomialtheorem. Usinginduction,thisproofwillcorroboratethetheorem forallintegers a ≤ 0

Firstandforemost,assignit p asaprimenumber. Thefirststepoftheproofisobvious 0p ≡ 0(modp),as 0 multiplyingordividingbyanynumberalwaysgives 0.Itisnowrequisitetoprovethatifthetheoremis trueforallothervaluesofa.Assumethestatement ap ≡ a(modp) istrue.Whenthebinomialtheoremis appliedfor (a +1)p ,

Accordingtotheoriginalbinomialtheorem,

Inthiscase,since p isaprimenumber, p!= p(p 1)!, and p divides p! ,butdoesnotdivide k! or (p k)! for 1 ≤ k ≤ p 1

Therefore, p k isdivisibleby p forall 1 ≤ k ≤ p 1

Furthermore,itcouldbestatedthatfor k =0 and k = p,1 p k equalsto 1.

Hence,theexpansionof (a +1)p simplifiesto

a +1)p ≡ ap +1(modp) (5)

As ap ≡ a(modp),itfollowsthat (a +1)p ≡ a +1(modp) (6)

Inessence,byinduction, ap ≡ a(modp) istruefor alltheintegersof a.

4FermatPseudoprimes

PseudoprimesareacaseofFermat’sLittleTheorem where p isacompositenumberintherelationap 1 ≡ 1(modp),where a and p arecoprimenumbers,and ap 1 1 isdivisibleby p.Ifthenumber p satisfies alltheconditionsabove, p iscalledaFermatpseudoprimetobase a.Forexample,let p intherelation be 341 341 isacompositenumberas 341=11x31 Normally,anycompositenumbersshouldfailtosatisfy Fermat’sLittleTheorem,asthetheoremisknownto betrueonlyforprimenumbers.Although 341 isnot truewhensubstitutedintherelation ap ≡ a(modp) → 3341 1 =3(mod341),itperfectlysatisfiestheotherrelation, ap 1 ≡ 1(modp).Therefore, 341 canbesaidthatit isacompositenumberaswellasaFermatPseudoprime.

5ConverseandCarmichaelNumbers

Longstoryshort,theconverseforFermat’sLittle Theoremdoesnotexist.Thisisduetoaparticular setofnumbersdubbedastheCarmichaelNumbers. Innumbertheory,aCarmichaelnumberisacompositenumber n inwhichmodulararithmeticsatisfies

Fermat’sLittleTheorem.Inotherwords,Carmichael numbersareatypeofpseudoprimes,which,whenpin thecongruencerelation, ap ≡ a(modp),isnotaprime number,unlikepreviouscaseswherepwasalways assumedtobeone;Carmichaelnumbersaretrue whensubstitutedintobothequationsofthetheorem. SomeexamplesofCarmichaelnumbersencompass 561(3 ∗ 11 ∗ 17), 1105(5 ∗ 13 ∗ 17),and 1729(7 ∗ 13 ∗ 19)

AlthoughFermat’sLittleTheoremcancorrectlydeterminetheprimalityforsomepseudoprimesasmostof themfailtosatisfybothrelationsofthetheorem,forexample, 341,thisisnotthecaseforCarmichaelnumbers. Unfortunatelyenough,Carmichaelnumberseasilysatisfybothrelationsofthetheorem,leadingtopotential inaccuraciesduringtheprimalitytestofthetheorem. Henceforth,themereexistenceofthissetofnumbers functionsasacounterexampleoftheclaimthatonly primenumberswouldsatisfyFermat’sLittleTheorem, becauseCarmichaelnumberswouldstillsatisfytherelationeventhoughtheyarenotprimenumbers.

6Implementations

AlthoughFermat’sLittleTheoremsporadically failsduringprimalitytests,itneverthelesscanbe utilisedtoaccuratelyderivetheremainderofacertaininteger.Usingthetheoremforexample,the remaindersforinconceivablylargenumberssuchas 39(299999986043 1) dividedby 299999986043,since 39 and 299999986043 arebothprimeandcoprimeto eachother,theremainderissimply, 1.

Furthermore,Fermat’sLittleTheoremplaysakey roleinmodernaffairssuchascryptographyandalso actedasacrucialpartinprovingtheoriesoffuture mathematicianssuchastheEulertheorem.

7Conclusion

Inconclusion,Fermat’sLittleTheorem,partofthe numbertheory,suggests2relations.Firstofall,

remaindersfornumbersincertaincircumstances.Fermat’sLittleTheoremhasbeenoneofthefoundational notionsofthenumbertheorywhichgreatlyinspiredand provokedthefieldofmathematicsbyencouragingfurtherresearchandaidingtocorroboratefuturetheories.

8Bibliography

1. Brilliant."Fermat’sLittleTheorem:BrilliantMath&ScienceWiki."Brilliant, brilliant.org/wiki/fermats-little-theorem/.

2. Caldwell,C.K."ProofofFermat’s LittleTheorem."ThePrimePages, t5k.org/notes/proofs/FermatsLittleTheorem.html.

3. "Fermat’sLittleTheorem."Artof ProblemSolving,artofproblemsolving.com/wiki/index.php/Fermat’s_Little_Theorem.

4. "Fermat’sLittleTheorem." WolframMathWorld,mathworld.wolfram.com/FermatsLittleTheorem.html.

5. GeeksforGeeks."Fermat’sLittleTheorem." GeeksforGeeks,geeksforgeeks.org/fermats-littletheorem/.

Essentially,itcouldbeprovenbyimplementingcombinatorics,thebinomialtheorem,andmany more.Althoughitpossessessomelimitationsregarding Carmichaelnumbersduringprimalitytests,thetheoremdemonstratesunparalleledpowerwhendetermining

ExploringtheEvolutionofGeometry

SoulPark Year11

Email:sopark27@pupils.nlcsjeju.kr

RecommendedYearLevel:KS4

Keywords:Geometry,Euclidean,Non-Euclidean, Hyperbolic,Riemann,Postulate

1IntroductiontoEuclideanGeometry

Geometryistheareaofmathematicsrelatedtothe studyofpointsandfiguresandtheirproperties.This fieldisessentialbecauseofitsundetachableapplication intherealworld.Euclid,theGreekmathematician alsoknownasthefatherofgeometry,foundedgeometry.

Euclidfurtherorganisedthemathematicalcontent inastrictlogicalsequencewithEuclid’scommon notionsandpostulates,takenfromtheElementsas follows:

CommonNotion:

1. Thingswhichequalthesamethingalsoequalone another.

2. Ifequalsareaddedtoequals,thenthewholesare equal.

3. Ifequalsaresubtractedfromequals,thentheremaindersareequal.

4. Thingswhichcoincidewithoneanotherequalone another.

5. Thewholeisgreaterthanthepart.

Postulate:

1. Astraightlinesegmentcanbedrawnjoiningany twopoints.

2. Anystraightlinesegmentcanbeextendedindefinitelyinastraightline.

3. Givenanystraightlinesegment,acirclecanbe drawnhavingthesegmentastheradiusandone endpointasthecentre.

4. AllRightAnglesarecongruent.

5. Iftwolinesaredrawnthatintersectathirdinsuch awaythatthesumoftheinneranglesononeside islessthantwoRightAngles,thenthetwolines inevitablymustintersecteachotheronthatsideif extendedfarenough.

Thefifthpostulateisequivalenttowhatisknown astheParallelPostulate,whichisthemostcontroversialoneandledtothedevelopmentofnon-Euclidean geometries.

2Non-EuclideanGeometry

Inthe19thcentury,theGreekgeometersviewed theparallelpostulateasatheoreminvolvingmany difficultiesasitsproofrequiredanumberofdefinitions andtheorems.ComparedtootherEuclid’spostulates, theparallelpostulatewasunclearandcomplicated.

Non-Euclideangeometryisusedtostatethetheory ofrelativityincurvedspaceandisappliedtocelestial mechanics.ThemajordifferencebetweenEuclidean geometryandnon-Euclideangeometryisdependenton thenumberofparallellines.Ifthereisexactlyoneparallelline,itisEuclideangeometry,andnon-Euclidean geometryiswhenthereisnoparallellineormorethan one.

WiththenumerousattemptsofProclus’andWallis’proofoftheparallelpostulatefromtherecognitionofthesignificanceofexponents,RussianNikolay LobachevskyandtheHungarianJánosBolyaipublished adescriptionofgeometrythatsatisfiedallofEuclid’s postulatesexceptfortheparallelpostulate,calledas thehyperbolicGeometry.

Fig.1:Non-EuclideanDiagram

3HyperbolicGeometry

In1868theItalianmathematicianEugenioBeltramidescribedasurfacecalledapseudospherethathas constantnegativecurvature.Thepseudosphereisnot acompletemodelforhyperbolicgeometry,asstraight linesmayintersectthemselves,unlikethehyperbolic postulate.Hence,in1901,theGermanmathematician DavidHilbertprovedthatacompletehyperbolicsurface cannotbedefinedusingrealanalyticfunctionsandwas notdiscussedfor54years.ThentheDutchmathematicianNicolaasKuiperprovedtheexistenceofacomplete hyperbolicsurface,andlaterinthe1970s,theAmericanmathematicianWilliamThurstondescribedhowit isconstructed.

Fig.2:CurvatureDiagram

HyperbolicGeometryisthefirstpublishedwork thatinterpretstheexistenceofhyperbolicandother non-Euclideangeometries.Theparallelpostulateof EuclideangeometryisreplacedwiththeHyperbolic parallelpostulate:

ForanygivenlineLandapointPnotonL,there existatleasttwolinesthroughPthatdonotintersectL.

Thekeypropertiesofhyperbolicgeometryincludes:

1. Thesumoftheinterioranglesofatriangleisless than180degrees.

2. Therearenosimilartrianglesofdifferentsizes

3. Linesthatappearparalleldiverges

ThiscontrastswiththeEuclideanpostulateasit statesthattwoparallellinesareequidistant.Unlikethe Euclidean,whenthesumoftheanglesinatriangleis equaltotworightangles;inhyperbolic,thesumisless thantworightangles.Consequently,polygonsofdifferingareascanbesimilarinEuclidean;andinhyperbolic, similarpolygonsofdifferentareasdonotexist.

Fig.3:EuclideanDiagram

Fig.4:HyperbolicDiagram

4HyperbolicModels

Later,hyperbolicmodelswerediscoveredinthe 19thcenturytoaidtheunderstandingofhyperbolicgeometryandvisualiseit.Thesemodelsareusedtoexpresstheanglesofparallelisminhyperbolicgeometry.

4.1Klein-BeltramiModel

From1869to1871BeltramiandtheGerman mathematicianFelixKleindevelopedthefirstcomplete modelofhyperbolicgeometry.Itisalsocalledtheprojectivemodel,Kleindiskmodel,andtheCayley–Klein modelinwhichpointsarerepresentedbythepointsin theinterioroftheunitdiskandthelinesarerepresented bythechords.

Fig.5:Klein-BeltramiModel

4.2PoincaréDiskModel

In1882,aFrenchmathematician,HenriPoincaré (1854-1912),studiedtwotypesofmodelsonthehyperbolicplane.Thefirstmodel,thePoincaréDisk,has overlappingsimilaritieswiththeKlein-BeltramiModel

yetwithasignificantadvantagethatmakesitmoresuitabletoworkwiththeEuclideananglebetweentwointersectinghyperboliclines.ItisconsideredastheimprovedmodelfromtheKlein-BeltramiModel.

Fig.6:PoincaréDiskModel

4.3PoincaréUpperHalf-PlaneModel

AnothermodelisthePoincaréhalf-planemodel, whichisthemostcommonlyusedmodel.Thismodel providesabettervisualisationcomparedtoothermodels,asitissuitableformorevariouscalculations.For thismodel,theEuclideanplanehasafixedlineinstead ofacircle,calledtheabsoluteline.Theabsolutelineis usedtorepresentthehyperbolicplane.

Fig.7:PoincaréUpperHalf-PlaneModel

5RienmannianGeometry

RiemanniangeometryoriginatedbyBernhard Riemann,aGreekmathematician,whofurtherextendedthenotionoftheparallelpostulateintotwoand three-dimensionalplanes.

InRiemanniangeometry,therearenolinesparallel tothegivenline.Euclid’ssecondpostulateis:thata straightlineoffinitelengthcanbeextendedcontinuouslywithoutbounds.However,Riemannmodifiesthe secondpostulate:astraightlineoffinitelengthcanbe extendedcontinuouslywithoutbounds,butallstraight linesareofthesamelength.

6Conclusion

Theevolutionofgeometryhasbeenstartedfrom thefoundationalworkofEuclidtothedevelopment ofthenon-Euclideangeometrieswiththeparallelpostulate.TheexplorationofthefifthpostulateofEuclid’sElementshasextendedtowardshyperbolicgeometry.Throughthediscoveryofhyperbolicgeometry, ithasemergedtofindexpressionsinmodelslikethe Klein-BeltramiandPoincarémodels.Furthermore,RiemanniangeometryexpandedupontheseideasbychallengingEuclid’ssecondpostulatewiththeideaofall straightlineshavingafinitelength.Thishighlightshow themathematicalapproachtounresolvableproblemsreshapestheunderstandingoffundamentalconceptsof geometry.

7Bibliography

1. Britannica.“HyperbolicGeometry.”Britannica, www britannica com/science/hyperbolicgeometry

2. Britannica.“Non-EuclideanGeometry.”Britannica, www britannica com/science/nonEuclidean-geometry

3. Britannica.“TheHistoryofHyperbolicGeometry.”Britannica, www britannica com/science/ hyperbolic-geometry#:~:text=The%20first% 20published%20works%20expounding

4. “Euclid’sElements.”OxfordReference, www.oxfordreference.com/display/10.1093/ acref/9780199235940 001 0001/acref9780199235940

5. Heath,T.H. TheThirteenBooksofEuclid’sElementsTranslatedfromtheTextofHeibergwith IntroductionandCommentary.DoverPublications,1956.GoogleBooks, books.google.co.kr/ books?id=HxhEBgAAQBAJ

6. “HistoryofHyperbolicGeometry.”MathosUniversity, www mathos unios hr/~mdjumic/uploads/ diplomski/PRO21 pdf

7. “HyperbolicGeometry.”StudySmarter, www studysmarter co uk/explanations/math/ geometry/hyperbolic-geometry/

8. “HyperbolicGeometryHistoryandApplications.”Study.com, study com/learn/ lesson/hyperbolic-geometry-historyapplications.html.

9. Milnor,John.“HyperbolicGeometryand ItsParallelPostulates.”RutgersUniversity, sites math rutgers edu/~cherlin/History/ Papers2000/eder html

10. Schombert,James.“HyperbolicGeometryinCosmology.”UniversityofOregon, pages uoregon edu/jschombe/cosmo/lectures/ lec15 html

11. Stillwell,John.“EuclidandBeyond.”Harvard University, people math harvard edu/~ctm/home/ text/class/harvard/113/97/html/euclid.html.

PrimeNumbersandGoldbach’sConjecture

wjhan28@pupils.nlcsjeju.kr

RecommendedYearLevel:KS4

Keywords:Goldbach’sConjecture,PrimeNumbers, Pairs

1Introduction

Primenumbersaremorethanjustabasiccuriosityfrommathematicians.Theyarethesolidfoundation formoderncryptography,computerscience,andeven physics.Fromprotectingonlineactionsperformedin ourdailylivestoconstructingmachinelearningalgorithms,primenumbersplayanessentialroleinourdigitalworld.Unfortunately,asoneofthemostdifficult andinterestingtopicsinmathematics,primenumbers havepuzzledmanymathematiciansthroughouthistory. TheseproblemsultimatelyincludeGoldbach’sConjecture,themostfamousandprominent.

2InfiniteStateofPrimeNumbers

BeforewetackleGoldbach’sconjecture,itisnecessaryforustodemonstratethatprimenumbersarein aninfinitestate,sinceiftheywerefinite,atsomepoint largeevennumberswouldlosetheirprimepairs,contradictingtheconjecture.Therefore,weuseEuclid’s Theoremofprimenumbers.

2.1Divergenceoftheharmonicseriesofprime numbers

ProvenbyEuler,itisstatedthatthesumofthe reciprocalsofallprimenumbersdiverges.

Iftherewereonlyafiniteamountofprimespresent, thissumwouldconvergetoacertainnumber.Forexample,let’ssupposethatthereareonly5numbersof primes:

Thesumoftheirreciprocalsarethefollowing:

Whichshowsthatthevalueconvergesatonepoint. However,ifmorevaluesofprimesareadded,

Thismeansthataswesummoretermsof1/p,the sumgrowscontinuously.

Asshown,thesevaluesneversettleonafinitevalue. Thegraphicalimplementationisshown:

Figure1showshowthecumulativesumofreciprocalsofprimesincreasesasmoreprimesareincluded. Itdemonstratesthatthesumdoesnotconvergetoa finitelimit;instead,itkeepsincreasingindefinitely, supportingEuler’sproofthatthereareinfinitelymany primes.

Fig.1:Cumulativesumofreciprocals

3Goldbach’sConjecture

Nowthattheinfinityofprimenumbershasbeen proven,wecannowsuggestwhetherthereisaninfinite setofevennumbersshownbyaddingappropriateprime numbersets.Inotherwords,theGoldbach’sConjecture statesthatanyevennumberNthatisbiggerthan2,can berepresentedbytwoprimenumberspandq,sothat

3.2Heuristicjustification

Goldbach’sConjecturecanbesupportedheuristicallyusingprobabilisticnumbertheory.Thecentral ideaisthatthenumberofwaystoexpressaneven integerNasasumoftwoprimescanbeestimated usingthePrimeNumberTheorem.

ThedensityofprimenumbersaroundNisapproximately: 1 ln(x)

Therefore,theprobabilitythatarandomlychosen numbernear N 2 isprimeisabout:

Forinstance,whenN=4,6,8,10,theycanbe representedas

4=2+2 6=3+3

8=3+5

10=3+7=5+5

Currently,thepatternhasbeentesteduptoatleast 4 × 1018,confirmingtheconjecturecomputationallybut notanalytically.

3.1Computation

Eventhoughtheconjectureisstillnotanalytically proven,thereareefficientalgorithmstoverifytheconjectureuptoaspecificpoint.Asimplebrute-forcealgorithmcanbeused:

Sinceweareselectingtwoindependentprimes,the expectednumberofprimepairsthatsumtoNcanbe approximatedby:

Whatthissumsuggestsisthatforasufficiently largeN,thereshouldbemanyvalidprimepairs,which makesancounterexamplehighlyunlikely.

Hereisagraphshowingthenumberofprimeparis forvariousevennumbersN:

Fig.3:y-axis:NumberofPrimepairs(p,q).x-axis: EvennumberN

Figure3showsthenumberofprimepairs(p,q)for eachnumberNupto2000.AsNincreases,thenumber

Fig.2:Abrute-forcealgorithm

ofvalidprimepairsalsogrows,whichsupportstheidea thateveryevennumberhasatleastonevalidprime pair.

3.3TwinPrimeConjecture

Goldbach’sconjectureiscloselyrelatedtothe TwinPrimeConjecture,whichstatesthatthereare infinitelymanyprimepairs(p,p+2).Thistheory strengthensGoldbach’sconjecture,sincetwinprimes canmakeupforGoldbachpairsforsmallevennumbers.

OneapproachtoprovingtheTwinPrimeConjectureinvolvestheHardy-Littlewoodconjecture,which predictsthenumberoftwinprimesuptox.

3.4Goldbach’sWeakConjecture

TheWeakGoldbach’sConjecturestatesthatevery oddnumbergreaterthan7canbeexpressedasthesum of3primenumbers:

4Conclusion

Goldbach’sConjectureremainsasanunsolvedmysterythathumanityshouldstillattempt.Although heuristicarguments,partialresults,andcomputation stronglysuggestitstruth,theproofisbeyondourreach. However,astheworldofmathematicsdevelopsandadvances,finalproofswillfittogether.

5Bibliography

1. rlagks05(2023).Exploringmathematicalconcepts relatedtoprimes.[online]NaverBlog.Availableat: https://m.blog.naver.com/rlagks05/223218483152

2. Wikipedia(n.d.).Goldbach’sconjecture-Heuristic justification.[online]Wikipedia.Availableat: https://en.wikipedia.org/wiki/Goldbach%27s_con jecture#Heuristic_justification

3. Wikipedia(n.d.).Divergenceofthe sumofthereciprocalsoftheprimes. [online]Wikipedia.Availableat: https://en.wikipedia.org/wiki/Divergence_of_the _sum_of_the_reciprocals_of_the_primes

Thisversionwasfinallyprovenin2013byHarald Helfgott,makingitonestepclosertoprovingthe conjecture.

Thistheorystatesthatthedensityofprimes ensuresthattheprobabilityofforminganoddnumber usingthreeprimesremainshigh.

Hereisahistogramshowingthenumberofwaysin whichdifferentoddnumberscanbeexpressedasasum of3primes.

Fig.4

Figure4showsthatthenumberofwaysdifferent oddnumberscanbeexpressedasthesumof3primes. Sinceasthenumbersincrease,thenumberoftriples grows,whichfurtherstrengthensGoldbach’sconjecture.

Tetrations

seohakim28@pupils.nlcsjeju.kr

Editor

RecommendedYearLevel:KS4,KS5

Keywords:Hyperoperations,Fractals,Spirals

1Introduction

Additionisconsideredtobeoneofthemostfundamentalyetarcaneideologiesinmathematicsthatformulatetheabsolutefoundationofthefield.Ostensibly, theoperatorisn’tregardedwithsuchprominencewithin modernsociety,butwithoutthefunctionsoftheadditionoperator,mathematicswillremainasunapproachabletheory,aninfeasibleconceptonlyeverdreamtof. However,thisisnotthecase,andmankindhassuccessfullyflourishedinmathematicssuchthatmodern technologyandscientificfeatswerenotpossiblewithouttheimplementationofthesubject.Theconceptto bepresentedinthisjournalisthe‘tetration’,anobscure operationnottaughtinschoolcurriculums.

2The4thHyperoperation

Weallhavebeenwellacquaintedwiththeaddition symbol(‘+’)eversinceourfirststepsinoureducational expedition.Thisisconventionallyreferredtoasthefirst hyperoperation;thus,thisimpliesthatmoreistofollow. Whenadditionisrepeatedlyperformedonanumber,we callitmultiplication(‘×’).Subsequently,theiterated performanceofmultiplicationresultsinthepowersymbol(‘bn’).Thecontinuationofthissequenceproceeds tobecomethefourthhyperoperation,officiallynamed ‘tetration’.Amereminorityisexposedtothissymbol, visuallyexpressedas nb.Thisoperationisanextensionofthesequentialprogressionoftheprecedingoperations.Inpracticalterms,3+4is7,3×4is3+3+3+3 whichequatesto12, 34 is3 ×3×3×3,whichresultsin 81,andultimately, 43 is 3333 ,whichroughlycalculates to1.25×103,638,334,640,024.Seeingasitexceeds3trilliondigits,nopracticalapplicationsseemplausibleat

thispoint.Thisforeignmathematicaloperationdoesn’t possessastandardnotationforexpression,andnounifiedsymbolexistsincasesforutilisationinproofs.The notation nbcanalsobefoundinthetransfiguredform ofb ↑↑ n.Thefactthatthesequenceofnumbersmoved from7to12to81toanumbernodifferenttoinfinity intriguedmathematicians,andeventuallytheyfounda patternthroughvisualisationofthegivendata.

3Implementation

3.1 ∞3

Fig.1:AVisualRepresentationoftheFourHyperoperations

Thefollowingisagraphshowingthevalueswhen eachofthefourhyperoperationswereperformedonthe number3(y=x+3,y=3x,y=3x,y= x3).The resultsareasexpected,andthefourlines,despiteall tendingtoinfinity,approachitatvariedrates,shown bythevariedgradients(andthevariedinstantaneous gradients)ofthegraphs.Thissetofgraphsisatypical exampleofthetetrationoperatorabidingbytheconventions.However,willthetrendbesustainedwhenthe

subjectofthegraphisalteredtoanirrationalnumber, say √2?

3.2 ∞√2

Fig.2:AVisualRepresentationoftheFourHyperoperations

Unsurprisingly,thefirstthreehyperoperationsdo notshowanydeviationfromthestandardconventions setinthepreviousgraph.However,thetetrationshows apatternwhichwaslaterfoundtotendto2; ∞√2, otherwiseknownas √2√2√2 ,hadavalueof2.Bothalgebraicandgraphicalmethodologiesexisttoprovethis.

Algebraically,toprovethat ∞√2 isrational(ideally 2),itiscrucialtoshowthatthesequence an= n √2 ismonotoneincreasing,i.e.strictlyincreasingand possessesnocrucialpoints,andalsoisboundedabove (meaningitconverges).Althoughitisalreadyshown throughthegreenlineinFigure2,algebraandsimple logicoutputtheconclusionthatsequentially,theupper boundis2.Regardingthetetrationofthesquareroot of2,thesequencewillbeassuch: √2, √2√2 , √2√2√2 Focusingonthesecondterm, √2√2 issmallerthan √2 2,justbythecomparisonoftheirindices.This means √2√2√2 willbesmallerthan2,sincetheindex ofthisnumber,asaforementioned,issmallerthan2, or √2 2.Thisiscarriedoncontinuously,eventually settlingupontheconclusionthatgiven ak=√2ak 1 , ak-1 willneverbegreaterthan2,andthusak will alwaysbelessthan √2 2.Theupperboundforthis sequenceisthereforesetto2.

Movingontothelatterportionoftheproof,let x=√2√2√2 .Throughsubstitution, x = √2x,and √2x x =0.Formulatinganequationregardingthis, f (x)= √2x x,and x isarootofthisequation.The relativemaximum/minimumcanbefoundbyderiving

theequationandsettingtheresultto0.Thestepsare asfollows:

f ′(x)= d dx 2 x 2 d dx (x)

f ′(x)=ln(2) × 2 x 2 × d dx x 2 1

f ′(x)=ln(2) × 2 x 2 × 1 2 × d dx (x) 1

f ′(x)=ln(2) × 2 x 2 × 1 2 × 1 1

f ′(x)=2 x 2 1 × ln(2) 1 (1)

Whentheequationissetto0,thex-valueofthe criticalpointis x = log√2( 2 ln(2) ),whichisapproximately3.06.Thenumericalvalueofthisequation doesnotpossessanysignificance;themerefactthat onerootexistsconveysgreatermeaning.Thus,itis foundthattheequationof x possesseseither2orno roots,butthroughobservation,itisevidentthat2 and4aretherootsforthisequation.Despitethis, asaforementioned, x isalwayslessthanorequalto 2,leadingtotheconclusionthattheanswerwouldbe2.

Agraphicalmethodthatrefrainsfromutilisingcalculusalsoexists.Thefollowingisagraphof y = x and y = √2x,alongwithadditionalguidelinestoassistthe explanation.

Fig.3:AGraphof y = x and y = √2x

Asclearlyshowninthegraphbelow,boththe x and y valuestendto2,whichistheconstantthat ∞√2 tendsto.Thiscontinuousstaircasewasconstructedby initiallysetting x to √2 andidentifyingthecorresponding y valueontheliney = √2x (colouredinred).These valuesarethevaluesof n √2 ,where n isanaturalnumberbeginningfrom1andsequentiallyincreasingasit tendstoinfinity.The x and y valueseventuallytendto 2,asshownintheintersectionbetweenthetwographs. Thiswasshownearlierwhentheequation x = √2x was introduced.Thus, ∞√2 is2.

3.3 ∞i

Whatwilloccurwhenthesubjectoftheoperation isalteredtoanimaginarynumber, i?Thevalueof i,whenprocessedwithtetration,producesvisually appealingresults.

Togetthebasicgistoftheoperation, 2i,or i i ,can becalculatedusingEuler’sidentity,oneofthemost renownedandeccentricformulasthatexistwithinthe fieldofmathematics,asitconnectsoneofthemostfundamentalrealandimaginarynumbersintoanequation. Now,proceedingwiththeproof,theEuler’sequation willbemanipulatedalgebraicallyassuch:

iπ +1=0

Finally,repeatingthetetrationwithanewvalue utilisingthesamemethodologies,itwasfound,after someresearchandexperimentation,thatthevalue 3i returnedresultsthatwerefairlyunexpectedanddeviatedfromthenorm.

Thefactthatthepowerofanimaginarynumberto animaginaryoneresultedinarealnumberastonished andpuzzledmany.However,whenthisprogressionis continued,agraphofextremeaestheticsandmathematicalsignificanceisproduced,asshownbelow.

i (y= x i)

Thefollowingcanbedescribedasaspiralfractal, wherethevaluestendto0.44+0.36i,aseeminglyarbitraryvalue.Thisspiralcouldalsobefoundinthenaturalenvironment,includingtheformationofthegalaxy. Additionally,suchpatternscanbefoundontheleaves ofplants,wheretheleavesformaspiralshapethatrolls continuouslyinwards.

∞3i

3i (y= x 3i)

Thefollowingwastheresultofthetetrationfor thevalue 3i.Initially,thefirstterm, 3i,wassolely imaginary,butasadditionaldatapointswereplotted, thegraphresembledatriangularshapethatperpetually repeated.Theverticesofthetrianglecorrespondedto thethreepoints0,1and0.08+3.06i.Duetothis,itis extremelychallengingtoidentifywherethegraphitself ‘tends’to,asnoendpointexists.Toberatherspecific, theendpointsdoexist,withtheproblembeingthatthe graphtendstohavethreepoints,whichwasnotthe caseforanyofthenumbersinvestigatedpriortothis one.

Throughtheseobservations,itbecomesclearthat

Fig.4:Alinegraphofthetetrationof
Fig.5:Ascatterplotofthetetrationof i (y= x i)
Fig.6:Alinegraphofthetetrationof

thevalueof ∞3i neitherconvergesnordivergesatone point.Keepinginmindthatthevalueof ∞ isconstant aslongasafinitenumberissubtractedfromit,itcan benotedthatifpoint 1 issetto ∞3i,thevalue 0 willbe ∞−13i and 0.08+3.06i as ∞−23i.Allvalues, ∞, ∞ -1 and ∞ -2areallnumericallyequivalentto ∞,meaning that ∞3i consistsofthethreevaluesaforementioned0,1and0.08+3.06i.Again,0.08+3.06iisyetanother arbitraryvaluethatpossessesnomathematicalsignificancewhatsoever.Withinthescientificrealm,thisis consideredstructurallysimilartotheideologiesinvolvingquantummechanics,whichdealswiththepossibility ofhavingmultipleoutputsforasingleinput.Suchare epitomesofhowmathematicscanoverrideconventions oflife.

4Fractals

Arecurringthemethatwasprevalentamongsttwo oftheexampleswerefractals,whichareself-repeating structureswhichtheoreticallyhavethepotentialto exceedtopologicaldimensionsandpossesscomplexity thatcannotbeillustratedthroughsimplesketchesand drawings.Suchfractalscanbeidentifiedwithinthe naturalworld,ascountlesslymentionedwithexamples likethegalaxy.Fractalscanalsobeobservedonplants andtheformationoftheirleaves.

Figure3showsastaircasefractal,whichrepeats continuouslyandgraduallydecreasesinsizeasittends toafinitepointonthegraph,2.Thisrepetitionwas formedbydrawingtheguidelinesinthegraph;the graphitselfismerelyacurve,alongwithastraight line.However,inthemidstoftheprocessofreaching 2,thisfractalwasformed.

Toelucidate,thepointbeganatthesquareroot of2,andthevalueof √2 inthegraph y = √2x ,or just y = √2√2 Withthesubstitution,thefirstoutput ofthetetrationwasrevealed 2 √2.Next,thehorizontal segmentofthestaircasemovedthe x valuesuchthat itisequaltothe y valuebyutilisingthegraphof y = x.Afterthis,theprocessisrepeated,suchthat x isnowthefirstoutputofthetetration, √2x ,and √2√2√2 .Theprocesseventuallysettlesatthepoint (2,2),suggestingthattheinfinitetetrationof √2 is2. Itwasfairlyintriguingthattheprocessofreachingthe answerresultedinafractal.

Theotherfractalformulawasshowninthetetrationoftheimaginarynumber i,onthecomplexplane. Thispatternthenrevealsthevaluesfor -1 i and 0 i as0 and1respectively,asmovingback,thespiralcanbe made.Thereareproofstowhyaspiralwasformed, andsuchproofsareintricatetoagreatextent.Thus, theunnecessaryexplanationswereomittedwithjust thecoreinformationremaining.

Notethatthegraphconsistsofa129º shiftateach point,formingaspiral.Anexact120º wouldresultina triangularshape,resemblingtheshapeshowninFigure 5.The9º offsetcreateddifferencesbetweeneverythird point.Additionally,ifalgebraicmeanswereutilisedto identifythepointofconvergence,theprincipalbranchof theLambertWfunctionwouldbeusedontheformula W = W ( ln(x)) ln(x) .Thisfunctioncanbeusedtoidentify theinfinitetetrationofanumber,andcanalsobeused toidentifywhetherthisvalueissignificantlyhigh(which suggeststhenumber’sinfinitetetrationdiverges).Based onthefollowing,thevaluefor i tetratedwas 0 438+ 0 361i,asidentifiedearlier.Thisformsaconverging spiral.

5FinalGraph

Shouldtherealmoftetrationceasehere,mathematicianswouldlosethepointinthepointinstudying thisoperation.Yet,onefinalgraphremainstoastound thosewhoseeit.

Asshownthroughthepreviousexamples,thereare caseswhentheinfinitetetrationofanumberresults intheconvergenceorthedivergenceoftheoutput. Whenthesevaluesarenotedandplottedonagraph dependingontheirattributesofbeingconvergentor divergent,therearepatternsthatarerevealed.Thisis knownasthetetrationfractal.

Duetothecharacteristicsofafractalbeinginfinite andrepeating,therearevariousshapesandpatterns thatcanresultfromzoominginandcapturingdifferent centresatvariedrangesofthe x and y values.The followingfractalbelowhasanupperthresholdofthe overflowlimitinPython,andwasformedfromthepoint (0,-150)to(150,150).

Admittedly,thefollowingisnotatypicalparadigm forthetetrationfractal,butcanalsobefound.For reference,atypicalfractallooksasfollows,andcontinuesinalldirectionseternally.Thefollowingimagewas madefromprofessionalgraphingsoftware,specialised forsuchtasks.

Fig.7:Ascatterplotofthetetrationfractal

Fig.8:Atypicalexampleofthetetrationfractal

6ComputerScience

Whenvisualisingdata,itisvitaltoensurethe dataismeaningful,andthatitcanbevisualised.All thefiguresillustratedaboveareuniqueproductions fromtheauthor,withtheexceptionofFigure8, whichwasfoundontheofficialwebsiteoftetration fractals.Figures1and2utilisedGoogleSheetsandits associatedfunctions,Figure3wasascreenshotfrom Desmos,agraphingcalculator,andFigures4to7were codedwithPython,ageneral-purposeprogramming language.Tomakevisualisationpossiblehere,this articleusedthenumpyandmatplotlibpackageofthe Pythonlanguage.Thecodeusedisasfollows:

Fig.9:Thegeneralcodeusedtodrawgraphs(Figs.4 6)

resultinaRecursionError,duetothelimitinthe maximumrecursiondepthofPythonfunctions.The basewaschangedbyadjustingthe‘1j’online8.Note that‘j’inPythonistheequivalentof‘i’inmathematics.

Movingon,thesecondsnippetofcodeusedthe sameprinciples,andalsohadatryandexceptfunction tocatchOverflowErrors.

Fig.10:Thecodeusedtocreatethetetrationfractal (Fig.7) 7Conclusion

Althoughthisarticleissolelyconsideredtetration, thereissuperfluouscontentregardingthefifthandsixth hyperoperations,anditcontinuessystematically.The materialintroducedinthispieceoftextwas,however, justameagreportionofthebiggerpicture,reservedfor thosewithgreaterinterestinthisfieldofmathematics. Itishopedthatthisarticleinspiresreaderstodofurther researchorfurtherstudies.

8Bibliography

1. Bourke,P.(2013).Tetrationfractal. [online]Paulbourke.net.Availableat: https://paulbourke.net/fractals/tetration/

2. Geisler,D.(2019).Tetration.org-Tetration.[online]Tetration.org.Availableat: https://tetration.org/original/Tetration/index.html

Tobrieflyexplainthefollowingcode,recursionwas usedtoraisethebasetothepoweroftheprevious term,withthepatternrepeated.Whenthetetration indexis1,thevaluewillalwaysbethebaseitself.The datawaslimitedto959points,as960orabovewould

3. SciPy(2008).lambertw—SciPyv1.15.0 Manual.[online]Scipy.org.Availableat: https://docs.scipy.org/doc/scipy/reference/generated/scipy.special.lambertw.html

RecommendedYearLevel:KS5

Derek(Yejun)Yoo Year12

Email:yjyoo26@pupils.nlcsjeju.kr

Keywords:Optimization,LagrangeMultiplier,NumericalMethods,PersonalFinance,DecisionTheory

1Introduction

Seollal,theKoreanLunarNewYear,isaculturallysignificanteventcelebratedbymillions.Itisatime whenfamiliescometogethertohonorancestors,share traditionalmeals,andexchangegoodwishes.Oneof themostanticipatedaspectsofSeollal,especiallyfor youngergenerations,isthecustomofreceivingmonetarygiftsfromelders.Thesefinancialgifts,oftengiven inneatlypackedenvelopes,provideanexcellentopportunityforrecipientstoengageinfinancialplanning.

Jeff,astudentwithakeeninterestinmathematics andfinance,findshimselfwithaconsiderablesumofSeollalmoney.Unlikemanyofhispeerswhomighthastily spenditongadgets,fashion,orentertainment,Jeffsees thisasanopportunitytoapplymathematicalprinciples tomakerationalfinancialdecisions.Thechallengehe facesisnotjustaboutspendingorsavingbutabout makinganoptimalallocationthatbalancesthreekey financialobjectives:security,growth,andliquidity.

Thisproblemnaturallylendsitselftothestudyof **constrainedmultivariableoptimization**,amathematicalframeworkusedineconomicsandfinanceto findthebestpossibleoutcomegivenspecificlimitations. Jeff’sgoalistomaximizehisfinancialwell-beingbydistributinghismoneyamongsavings,investments,and immediateexpenditureswhileadheringtoreal-worldfinancialconstraints.

2MathematicalFormulationoftheProblem Jeffdecidestodistributehismoneyacrossthreeprimarycategories:

1. Savings (x):Alow-riskaccountofferingstablebut modestreturns.

2. Investments (y):Stocksorotherhigh-returnfinancialinstruments,whichcomewithanassociated levelofrisk.

3. ImmediateSpending (z):Thiscoversday-to-day expensesanddiscretionarypurchasesthatofferinstantutility.

ThetotalutilityJeffderivesfromhismoneyisrepresentedbythefunction:

where:

Theterms 0 03x and 0 08y reflectlinearreturnson savingsandinvestments,respectively.

Thequadraticterm 0 01y2 modelsdiminishingreturnsoninvestments,capturingtheideathathighriskassetsdonotalwaysyieldproportionalrewards. Thelogarithmicterm 0 5ln(z) ensuresthatspendingcontributestoutilitybutexhibitsdiminishing marginalbenefits.

Jeffalsofacesseveralreal-worldconstraints:

x + y + z ≤ 300 (Totalbudgetconstraint)(2)

0 05y ≤ 10 (Riskconstraint,limitingexposuretovolatileassets) (3)

z ≥ 50 (Minimumliquidity,ensuringhehasenoughcashforessentials) (4)

3OptimizationUsingLagrangeMultipliers

TheLagrangemultipliermethodisapowerful mathematicaltechniqueusedtofindthelocalmaximaandminimaofafunctionsubjecttoequalityconstraints.InJeff’sscenario,heaimstomaximizehis utilityfunction U (x,y,z),representingthesatisfaction derivedfromallocatinghisSeollalmoneyintosavings (x),investments(y),andimmediatespending(z).This optimizationissubjecttoconstraintssuchashistotal budgetandrisktolerance.

ToapplytheLagrangemultipliermethod,wefirst definetheconstraintfunction g(x,y,z) thatrepresents thelimitationsJefffaces.Forinstance,abudgetconstraintcanbeexpressedas:

TheLagrangianfunctionisformulatedas:

L(x,y,z,λ)= U (x,y,z) λ(x + y + z 300) (5)

WethenconstructtheLagrangianfunction, L, whichcombinestheoriginalutilityfunctionandtheconstraintusinganewvariable λ (theLagrangemultiplier):

L(x,y,z)= U (x,y,z) λg(x,y,z) (6)

Thenextstepinvolvestakingthepartialderivatives of L withrespecttoeachvariable(x,y,z)andtheLagrangemultiplier λ,andsettingthemequaltozero:

Step1:DefinetheProblem Jeff’sutilityfunctionwasgivenby:

Solvingthissystemofequationsyieldsthevaluesof x,y,z, and λ thatmaximizeJeff’sutilitywhilesatisfying theconstraint.

Geometrically,thismethodensuresthatthegradientoftheutilityfunction ∇U isparalleltothegradientoftheconstraintfunction ∇g attheoptimalpoint. Thismeansthatthedirectionofthesteepestascentof theutilityfunctionisalignedwiththedirectionofthe constraintsurface,ensuringthatanymovementalong theconstraintdoesnotincreasetheutility,indicating anoptimalallocation.

TheLagrangemultiplier λ itselfprovidesvaluable economicinsight;itrepresentstherateatwhichthe maximumvalueoftheutilityfunctionincreasesasthe constraintisrelaxed.InJeff’scase, λ indicateshow muchhismaximumutilitywouldincreaseifhisbudget constraintwereslightlyexpanded,providingameasure ofthemarginalutilityofadditionalresources.

ByapplyingtheLagrangemultipliermethod,Jeff cansystematicallydeterminetheoptimalallocationof hisSeollalmoneyacrosssavings(x),investments(y), andspending(z),ensuringthatheachievesthehighest possiblesatisfactionwithinhisfinancialconstraints.

Withtheconstraintbeing:

TosolvethisusingtheLagrangemultipliermethod, wecombinetheutilityfunctionandtheconstraintinto asinglefunction,theLagrangian:

Step2:ComputethePartialDerivatives

Wecalculatethepartialderivativesof L withrespecttoeachvariable(x,y,z,and λ).

1. Withrespectto x: δL δx =0 03 λ =0 (11)

2. Withrespectto y: δL δy =0 08 0 02y λ =0 (12)

3. Withrespectto z: δL δz = 0 5 z λ =0 (13)

4. Withrespectto λ: δL δλ = (x + y + z

Step3:SolvetheSystemofEquations

Thesystemofequationstosolveis:

1. λ =0 03 (from δL δx =0)

2. λ =0.08 0.02y (from δL δy =0)

3. λ = 0 5 z (from δL δz =0)

4. x + y + z =300 (fromtheconstraint)

Step4:SubstitutingValues

1. Fromequation(1),substitute λ =0 03 intoequation (2): 0 03=0 08 0 02y (15)

Solvingfor y: y = 0.08 0.03 0 02 =2 5 (16)

2. Substitute λ =0 03 intoequation(3): 0 03= 0 5 z (17)

Solvingfor z:

3.Usethebudgetconstrainttosolvefor x:

(18)

Step1:ComputetheGradientof U (x,y,z) Thegradientof U (x,y,z) isthevectorofpartial derivatives:

U (x,y,z)= δU δx , δU δy , δU δz

Forthegivenutilityfunction:

Step2:InitializeStartingValues

Chooseinitialguessesfor x,y,z: x0 =100,y0 =100,z0 =100

Then,setalearningrate(α)tocontrolthesizeof eachstep.Forthisproblem,let α =0 1

Step3:IterativeUpdates

Ateachiteration,update x,y, and z usingthegradientascentformulas:

x +2 5+16 67=300 x =300 2.5 16.67=280.83

FinalSolution

x =280 83,y =2 5,z =16 67,λ =0 03

4NumericalOptimizationApproach

Method2:NumericalOptimizationusingthe Gradient-basedMethod

Numericaloptimizationisapracticalandpowerful approachtosolvingconstrainedoptimizationproblems, particularlywhenanalyticalmethodsliketheLagrange multiplierarechallengingorinfeasible.Unlikeanalyticalmethods,whichaimtofindanexactsolution,numericaloptimizationiterativelysearchesfortheoptimal solutionbyapproximatingthemaximumorminimum oftheobjectivefunction.InJeff’scase,numericaloptimizationhelpsdeterminethebestallocationofhisSeollalmoney(x,y,z)acrosssavings,investments,andimmediatespending,subjecttobudget,risk,andliquidity constraints.

Afterupdatingthevalues,checkwhetherthedata constraintsaresatisfied:

1. If x + y + z> 300,adjustthevariablestoensurethe budgetconstraintismet.

2. If y> 200,set y =200

3. If z< 50,set z =50.

Step4:Convergence

Repeattheupdatesuntilthechangesin x,y, and z betweeniterationsareverysmall(e.g.,lessthan0.01), indicatingthatthesolutionhasconvergedtotheoptimalallocation.

WhyNumericalOptimization?

1. Handlesnon-linearfunctionslike U (x,y,z) effectively.

2. Accommodatescomplexconstraintsinastraightforwardmanner.

3. Providesapproximatesolutionswhenexactanalyticalsolutionsareimpractical.

1

ForJeff,thismethodnotonlyfindsthebestallocationofhismoneybutalsohighlightstheroleof constraintsinshapinghisdecision-makingprocess.By iteratingtowardtheoptimalsolution,numericaloptimizationoffersaclear,adaptableapproachtoreal-world problems.

Method3:Python

2 # Define the utility function and its gradient

3 def utility(x,y,z):

4 return 0.03*x+0.08*y-0.01*y**2+ → 0.5*np.log(z)

5

6 def gradient(x,y,z):

7 grad_x=0.03 # Partial derivative with → respect to x

8 grad_y=0.08-0.02*y # Partial → derivative with respect to y

9 grad_z=0.5/z # Partial derivative with → respect to z

10 return np.array([grad_x,grad_y,grad_z])

11

12 # Gradient ascent with constraints

13 def optimize_budget(initial_x,initial_y, → initial_z,budget,alpha,max_iter, → tolerance):

14 x,y,z=initial_x,initial_y,initial_z

15 for in range (max_iter):

16 # Calculate gradient

17 grad=gradient(x,y,z)

18

19 # Update variables using gradient → ascent

20 x_new=x+alpha*grad[0]

21 y_new=y+alpha*grad[1]

22 z_new=z+alpha*grad[2]

23

24 # Apply constraints

25 if x_new+y_new+z_new>budget: # → Budget constraint

26 excess=(x_new+y_new+z_new)→ budget

27 total=x_new+y_new+z_new

28 x_new-=(x_new/total)*excess

29 y_new-=(y_new/total)*excess

30 z_new-=(z_new/total)*excess

31

32 y_new= min (y_new,200) # Risk → constraint

33 z_new= max (z_new,50) # Liquidity → constraint

34

35 # Check convergence

36 if abs (x_new-x)<tolerance and abs ( → y_new-y)<tolerance and abs ( → z_new-z)<tolerance:

37 break

38

39 # Update variables for the next → iteration

40 x,y,z=x_new,y_new,z_new

41

42 return x,y,z

43

44 # Parameters for optimization

45 initial_x=100

46 initial_y=100

47 initial_z=100

48 budget=300

49 alpha=0.1

50 max_iter=1000

51 tolerance=0.01

52

53 # Solve for optimal allocation

54 optimal_x,optimal_y,optimal_z= → optimize_budget(initial_x,initial_y, → initial_z,budget,alpha,max_iter, → tolerance)

55

56 optimal_x,optimal_y,optimal_z

Listing1:GradientAscentOptimizationforBudget Allocation

5Conclusion

Jeff’sfinancialdilemmaisaclassicexampleofconstrainedoptimizationinaction.Whetherusing**Lagrangemultipliers**or**numericaloptimization**, thiscasestudydemonstrateshow**mathematicsprovidesastructuredapproachtofinancialdecisionmaking**.

PoissonDistributionandHowIt’sUsedinSports Betting

Email:hj2lee26@pupils.nlcsjeju.kr

RecommendedYearLevel:KS4+

Keywords:Poissondistribution,Numberofsuccess,Probability

1Introduction

Weoftenseepeoplebettinginsports,withmanypeopleattemptingtopredicttheoutcomeofmatches,oftenrelyingheavilyonintuitionandpersonalbias.Thesesubjective guesseseasilyleadtobettingaddictionandmassivefinanciallosses.Whatiftherewereawaytomakemoreinformed andreasonedpredictionsthroughmathematicalanalysis?By leveragingdataandstatisticalmodels,itbecomespossibleto enhancetheaccuracyofsportspredictions,reducingreliance ongutfeelings.

2IntroductiontoPoissondistribution

Forourprediction,weturntofootballdataandapply thePoissondistribution.ThePoissondistributionisaprobabilitydistributionthatpredictshowmanytimesaneventis likelytooccurwithinagiventimeperiod.ForarandomdiscretevariableXthatfollowsthePoissondistribution,andis theaveragerateofvalue,thentheprobabilityofXtimesof eventsisgivenby:

x=numberofsuccesses

λ =rateofsuccess

e=Euler’snumber(2.71828)

Inthiscontext,successreferstoscoringagoal,andthe rateofsuccessrepresentstheaveragenumberofgoalsscored byateampermatch.

APoissondistributioncanbeappliedunderthefollowingconditions:

1.Eachevent(goal)occursindependently,meaningthe likelihoodofonegoaldoesnotaffecttheprobabilityof another.

2.Theaveragenumberofeventsoveraperiodisconstant andknown(representedby λ (lambda)).

3ManchesterCityvManchesterUnited

TodemonstratetheapplicationofthePoissondistribution,let’sanalyzeamatchupbetweenManchesterCityand ManchesterUnitedinthePremierLeague.Byexamining theirhead-to-headrecordsince2017,wecanpredictpotentialoutcomesoftheirnextmatch.

MCI MNU

GamesPlayed 22 22 Goalspergame 38 26

Withtheaveragegoalspergamecalculated,wecanapplythePoissondistributiontoestimatetheprobabilityof eachteamscoringaspecificnumberofgoals(e.g.0to5 ormore).

Thevisualizationsofcalculationcarriedoutforeach teamarethefollowing.

Figure1.PoissonDistributionoftheScoringProbabilitiesofManchesterCity

Figure2.PoissonDistributionoftheScoringProbabilitiesofManchesterUnited

Then,Aheatmapcanbecreatedtovisualizeallpotential scorecombinations,showingtheprobabilityofeachpossible outcome.Eachboxontheheatmaprepresents,ofcourse, theproductofthegoalprobabilitiesforManchesterCityand ManchesterUnited.

Figure3.AheatmapofpossibilitiesforthepossiblescorecombinationsbetweenCityUnited

Thevisualizationgenerallyrevealsthatlow-scoring games(0-2goalsperteam)arethemostlikelyoutcomes. TheheatmapalsoindicatesthatManchesterCityisstatisticallymorelikelytowin,withthesecondmostprobable outcomebeinga2-1winforCity.

Basedonthisanalysis,themostrationalbetswouldfocusona1-1drawora2-1winforManchesterCity,ifyou alsogobythecommonsensethatCityisobjectivelyabetter teamthanUnited.

4Extension

GiventhatthePoissondistributioncandealwiththe numberofoccurrencesinagiventimeperiod,itisalsopossibletocalculatethenumberofgoalsinthefirsthalfandthe separatehalf.Thiscanbehelpfulinevaluatingtheoffensive consistencyofteamsacrossthefullgame,allowingforbetter strategicdecisions.

Additionally,itcanpredictthetotalnumberofgoalsa teammayscoreoverthecourseofa38-gamePremierLeague season.

Byapplyingthesamemethodology,predictionscanbe extendedtoindividualplayers’goal-scoringorassiststatistics.Althoughattackersnaturallyhaveahigherprobability ofscoring,thismightcauseissuessuchastheattackerbeing thefavoritetoscoreallthetime,whichiscertainlynotthe caseinreality.However,thePoissonmodelself-corrects.If anattackerfailstoscore,theirsuccessrate()decreasesover time,redistributingtheprobabilitytootherplayers,leading toamoreaccuratemodel.

5Limitations

However,stillthisisafairlyunprofessionalandoversimplifiedanalysisofafootballgamethatignoresmany importantfactors,suchasthesquadquality,injuries, home/awayadvantage,orhumanitarianelementssuchas playerfatigueorintenseschedules.

Themodelreliesprimarilyonpastperformanceand assumesthathistoricalgoaldatareflectsthevariableslike theonesabovethatcannotbeexplicitlymeasured.Consequently,whilethePoissondistributionoffersausefultoolfor prediction,itshouldbecoupledwithamuchbroaderanalysis.

6Conlcusion

ThePoissondistributionprovidesasimpleapproachto predictingsportsoutcomes.Althoughthismethodsimplifiesmanycomplexitiesofreal-worldsports,itopensthedoor forfurtherrefinementandintegrationwithadditionalpredictivefactors,suchasplayerform,injuries,andotherdynamic variables.Whilenotfoolproof,Poissondistributionoffersa valuablefoundationformoreinformedsportspredictions.

Bayes’TheoremandPsychology:AreOur PerceptionsofProbabilityMisguided?

Suhyuk(Jeff)Cho Year12

Email:shcho26@pupils.nlcsjeju.kr

Editor

RecommendedYearLevel:KS4,KS5

Keywords:Conditionalprobability,Representationof probabilities,Bayes’Theorem,Psychology

1Introduction

Eveninyourmostmundaneday,youarelikely toencountertheconceptofprobability,regardlessof whetheryouareconsciousofit.Youunconsciously makenumerousdecisionsbasedontheprobabilityand likelihoodofdifferentevents,whicharenaturallycalculatedinsideyourbrain.Butwhenpeopleinitially comeupwithahypothesisandthenareintroduced tonewinformation,quiteoftenthisleadstoanirrationaldecision,heavilyinfluencedbythelatestknowledge.Inotherwords,ratherthan‘updating’thehypothesisheldbeforebeinggiventheinformation,peopledisregardorprioritisethenew,whichmakesthem makeachoiceratherthanirrational.TheBayes’theoreminsteadshowshowtheprobabilitychangesgivena certainconditionandhowtheoutcomecandifferfrom theinitialhypothesis.Itallowspeopletodeterminethe conditionalprobabilityofanevent.

2Bayes’theorem

2.1Thebasics

Considerthisverybasicexample:Infrontofyou are50vehicles,whichareeitheracaroramotorcycle. Outofthe50vehicles,30arecars,and20aremotorcycles.

Arichmancomesbyandsaysthathewantsto randomlyselectavehicleandbuyit.Whatisthe probabilitythathewillbuyacarorthathechoosesa motorcycle?

Fig.1:Case1

Theanswerisverysimple.Sincethereare30cars and20motorcycles,theprobabilityofchoosingacaris 60%whilechoosingamotorcycleis40%. Fig.2:Simplerepresentationofcase1

Butthistime,outofthe30cars,15areblue,and outofthe20motorcycles,only5areblue.

Therichmannowwantstobuyabluevehicleand hewantstodothisrandomly.Doestheprobabilityof selectingacarorselectingamotorcyclechange?

Fig.3:Case2

Yes,itwillchange.Duetothisnewrestriction,the probabilityofchoosingacarincreasedto75%whilethe probabilityofchoosingamotorcycledecreasedtoonly 25%.

Fig.4:Simplerepresentationofcase2

Whenweweremakingthiscalculationweonlyconsideredadescriptiongivenfromthepoolofbluevehicles thatrestrictedthenumberofcarsormotorcyclesthat hadthepossibilityofbeingchosen.

2.2Case-specificformula

Liketheaforementionedexample,ifnewinformation(bluevehicle)isintroducedtoaninitialhypothesis (60%car:40%motorcycle),theoverallprobability canchangeandthereforea’update’tothepreliminary probabilityshouldbemadeinaccordancewiththis.

Whengiventheinitialhypothesisthatoutof50 vehicles,30arecarsand20aremotorcycles,theprobabilityofrandomlyselectingacarcanbecalculatedby theformula

p(c)= Numberofcars n (1)

Thisgoessamewiththemotorcycles.

p(m)= Numberofmotorcycles n (2)

wherenisthetotalnumberofvehicles

So,ifweformanewformulabasedonthe‘update’ thatthereare15bluecarsand5bluemotorcycles,and

trytoworkouttheprobabilityofselectingabluecar outofallbluevehicles(condition),

where

p(b|c)=Theprobabilityofchoosingabluecarout ofallcars

p(b|m)=Theprobabilityofchoosingablue motorcycleoutofallmotorcycles

Inotherwords,theprobabilityofchoosingablue carisequaltothenumberofbluecarsoverthenumber ofbluecarsaddedtothenumberofbluemotorcycles (whichisultimatelythenumberofbluevehicles).

Tohelpwiththeunderstanding,theformulaabove isthesameas

p(c|b)= Numberofbluecars Numberofbluecars + Numberofbluemotorcycles (4)

Fromhere,wecanobtainthefollowingvalues:

n=50(vehicles)

p(c)=0.6

p(m)=0.4

Sincethereare15bluecarsoutofall30cars, p(b|c)=0.5

Sincethereare5bluemotorcyclesoutof20all motorcycles, p(b|m)=0.25

Ifwesubstituteinthesevalues, p(c|b)= 50 0.6 0.5 50 0 6 0 5+50 0 4 0 25 =0 75 =75% (5)

2.3GeneralFormula

Amoregeneralformulawithstandardisednotations canrepresentasimilarsituation.Rememberthatweare usingtheconceptofhypothesis(representedasH)and

Evidence(meaningthenewlyintroducedinformation, representedasE).

p(c|b)= n (H) P (E|H) n (H) P (E|H)+ n P (¬H) P (E|¬H) (6)

where

p(H|E)=Theprobabilitythatthehypothesis holds given thattheevidenceistrue(thisisthesame astheprobabilityofchoosingabluecaroutofall blue vehicles)

n=Thetotalnumberofcases

p(H)=Thepriorprobability(notgiventhenew evidence)ofthehypothesis

P(E|H)=Theprobabilitythattheevidenceholds given thatthehypothesisistrue(theprobabilityof choosingabluecaroutofallcars)

¬H=Whenthehypothesis isn’t true(themotorcycles,sincetheyarenotcars)

Thetotalnumberofvehicles n canbedividedfor boththenumeratorandthedenominator.Fromhere, theexpressionleftinthedenominatorcanbereplaced byp(E),since

p(E)=Thetotalprobabilityofseeingtheevidence (theprobabilityofchoosinga bluevehicle)

Hence,amoresimplifiedformulaoftheBayes’theoremwouldbe

portfolios.Numerousfactorsareaccountedforin thecalculations,hencetheyallcorrespondtothe ‘evidence’oftheBayes’theoremwhichmodifiesthe probabilities.

3. MedicalDiagnosis:Basedonpriorknowledge andtestoutcomes,theprobabilityofbeingacertain diseaseoraconditioncanbecalculatedbymatching withthepatient’ssymptomsandtestresults.

3Linktopsychology

3.1KahnemanandTversky’sexperiment In2011,DanielKahneman,awidelyacclaimedpsychologist,publishedthebook Thinking,FastandSlow whichdiscusseshumandecision-makingandrationality basedonhissetofpsychologicalexperimentsthat showedthathumanintuitionisimperfect.Heclaimed thatthechoicesandjudgementsahumanmakescan deviatefromthenormativestatisticalmodels,which canoftenresultinirrational,nonstatisticalandflawed decisions.

OneveryfamousexperimentKahnemanandhis partnerAmosTverskyconductedwastogiveabrief descriptionofamannamed Steve

“Steve isvery shyandwithdrawn,invariably helpfulbutwithverylittleinterestinpeopleorthe worldofreality.A meekandtidy soul,hehasaneed fororderandstructure,andapassionfordetail.Is Stevemorelikelytobea librarian ora farmer ?”

Afterhearingthisdescription,mostansweredthat Steve ismorelikelytobea librarian.Whenquestioned why,thetwomostpopularreasonswerethetwo descriptivephrasesabouthim:‘shyandwithdrawn’ and‘meekandtidy’.Sincethesedescriptionsarelikely tobeassociatedmorewithastereotypicallibrarian thanafarmer,thisisdirectlyajudgementthat Steve ismorelikelytobealibrarian.

2.4Applications

Albeititssimplisticnature,Bayes’theoremisfrequentlyusedinvariousfieldsandistreatedasthe basisfordeterminingandcalculatingcomplexconditionalprobabilities.Somenotableapplicationsofthe Bayes’theoremareasfollows.

1. Spamfiltering:Althoughtheprocessrequires greatercomplexity,thebasisofidentifyingandremovingspamisbyspottingtheoccurrenceofcertainwordsorphrases,andthencalculatingthelikelihoodoftheemailbeingspambasedonthetrained dataset.

2. Financialforecasting:Historicmarketdataand economicindicatorsareanalysedtomatchand projectthepossibleoutcomeofanyinvestments, whicharetheninreturnusedtocreateinvestment

Butthisdecision,accordingtoKahneman,is irrational.Nearlyeveryonewhoparticipatedinthe experimentdisregardedthefactthattherearesignificantlymorefarmersthanlibrariansintheUnited States(wheretheexperimentwasconducted).The ratiobackthenwas20:1,whichshowsthatAmerican citizenswouldhavealreadyknownthedifferenceinthe numberoffarmersandlibrariansbutjustproceededto ignorethisknowledge.

Here,torationallydeterminethelikelihoodof Steve beingeitherafarmeroralibrarian,Bayes’theorem canbeused.The‘hypothesis’wediscussedbeforeis nowthepreconditionthatthereexist20timesmore farmersthanlibrarians,andthe‘evidence’whichalters theprobabilityisthelikelihoodofalibrarianora farmerbeing‘shyandwithdrawn’and‘meekandtidy’.

Whileitistruethatthesequalitiesarestereotypically associatedwithalibrarianmorethanafarmer,the initialhypothesismakesitextremelydifficultfor Steve tobemorelikelytobealibrarian.

For Steve,ameekandtidyman,tohaveanequal probabilityofbeingeitherafarmeroralibrarian,

Ifweassumethat all librariansare‘meekandtidy’, andthenumberof‘meekandtidy’farmersis x,

“Lindaisthirty-oneyearsold,single,outspoken, andverybright.Shemajoredinphilosophy.As astudent,shewasdeeplyconcernedwithissuesof discriminationandsocialjustice,andalsoparticipated inanti-nucleardemonstrations...”

Therespondentswereaskedtoranktheorderof likelihoodofLindabeing:

a)anelementaryschoolteacher

b)activeinthefeministmovement

c)abankteller

d)aninsurancesalesperson

e)abanktelleralsoactiveinthefeministmovement

Quitesurprisingly,85%oftherespondentsranked e)higherthanc).Althoughatfirstglancethismight feelreasonable,thiscompletelyignoresthebasicrules ofprobability:e,abanktelleralsoactiveinthefeminist movement,isasubsetofc,abankteller.Inotherwords, eventhoughafeministbanktellerstillisabankteller, mostpeoplemadetheirrationaldecisionofrankingit aboveabroaderterm.

Hence,whatthismeansisthatevenundertheextremecircumstancethatalllibrariansintheUnited Statesare‘meekandtidy’,for Steve tohavea50:50 chanceofeitherbeingalibrarianorafarmer,there shouldexistatmostonefarmerforeverytwentyfarmerswhoisrelevanttothedescriptionof Steve.Which, consideringthevariouspersonalitiesofdifferentpeople, isalmostimpossible.

3.2Humanmindandprobability

Atthispoint,onemightquestionwhetherthis psychologicalexperimentbecomesrelevantonlyand iftherespondenthaspriorknowledgeoftheratio betweenthenumberoflibrariansandfarmersinthe UnitedStatesatthattime.However,thisisnotthe focusofthisexperiment.Themostimportantpartis whethertheparticipanthasevenconsideredusingthis informationtoformhis/herjudgement.Withouteven accountingforapieceofinformationthatcanseverely changetheanswer,itiscorrecttosaythatinmaking decisionsforthesetypesofquestions,we,ashumans, areirrational.

Butdowealwaysmakeirrationaldecisions?Or, aretherecertainmomentswherebiaseskickinwhen makingdecisions?

Onewayourintuitionofprobabilitychangesisin thelanguageandhowwerepresentaprobability.

Considerthisexamplefromanotherexperiment conductedbyKahnemanandTversky.

Fig.5:Venndiagramofthetwooccupations

Butwhenthequestionwasrephrased,noonemade thesamemistake.

“100peoplefitthisdescription.Howmanyare:”

a)Banktellers?_outof100

b)Banktellersandactiveinthefeministmovement?_outof100

Eachrespondentputalargernumberforathan b,whichwerecorrectdecisionstakingintoaccountthe factthatfeministbanktellersareasubsetofbank tellers.

Althoughwecananalysethisdrasticdifference injudgementsusingvariouspsychologicalfactorslike whatKahnemanandTverskydid,asimplerfocuscan bemadetowardshowprobabilityisrepresentedineach case.

Itisundeniablethatgraphicalmethodsofrepresentingprobabilitiesarebyfarthemostintuitive

method,andbyusingthis,itisveryunlikelythata personwouldfallintothe‘trap’likewhatKahneman andTverskyhadsetupforbothexamples Steve and Linda.Onthecontrary,thereisatendencyforphrases thatdonotincludeanydefinitenumericalvalueslike ‘morelikely,’,or‘haveagreaterlikelihoodof...’to causepeopletohaveblindnessoveranystatistical factorsthatshouldhavebeentakenintoaccountto maketheirdecisions.

Afterall,anycalculationsorevenroughestimates givenaconditionalprobability(whichrequiresBayes’ theoreminourheads)aremadeafterrealisingboththe ‘hypothesis’andthe‘evidence’,notwhenthefocusis madeononlyoneofthem.

4Conclusion

Throughthisarticle,wefirstinvestigatedwhat Bayes’theoremisanditsapplications,withanextensionintotherelationshipbetweenthehumanmind andprobability.AlthoughBayes’theoremlookscomplicatedfromtheoutside,itisasimplecalculationwe makeinourheadswhenencounteringanyconditional probability.Butassaidinsection3,thesecalculations donotoccurwhenwecannotrecognisethatthereare boththehypothesisandtheevidencetoaproblem, withsometimesonehiddeninthephrasingwhich hindersusfrommakingrationaldecisionsintheend.

Hence,thenexttimeyouencounteranyproblems orsituationsthatsuspiciouslyguideyoutowardsone choice,rememberthatyoumightberestrictingyourselffromthebroaderperspectivethatcancompletely transformyourfinalchoice.Youdon’twanttorepeat thesamemistakeyouhavemadefor Steve!

5Bibliography

1. StanfordEncyclopediaofPhilosophy(2003).Bayes Theorem.[online]plato.stanford.edu.Availableat: https://plato.stanford.edu/entries/bayes-theorem/ 2. Tversky,A;Kahneman,D.(1974).JudgmentunderUncertainty:HeuristicsandBiases.[online]psych.ubc.ca.Availableat: https://www2.psych.ubc.ca/schaller/Psyc590Re adings/TverskyKahneman1974.pdf

3. Kuter,K.(2021).ConditionalProbabilityand Bayes’Rule.[online]libretexts.org.Available at:https://stats.libretexts.org/Courses/SaintMary’s-College-Notre-ame/MATH-345Probability(Kuter)/23A-ComputingProbabilities/2.23A-Conditional-Probabilityand-Bayes’-Rule

CryptographyandFermat’sLastTheorem

Minseong(Eric)Ko

Year10

Email:msko28@pupils.nlcsjeju.kr

Editor Junseok(Jayden)Lee

RecommendedYearLevel:KS4

Keywords:Fermat’sLastTheorem,NumberTheory, Cryptography

1Introduction

Onemighthavewonderedhowmathematicsisused incryptography.Inordertoknowthat,thestorybegins 400yearsago.

“Itisimpossibleforanynumberwhichisapower greaterthanthesecondtobewrittenasasumoftwo likepowers.Ihaveatrulymarvelousdemonstration ofthispropositionwhichthismarginistoonarrowto contain.”Thisenigmaticwordscribbleddownin1637by PierredeFermatgaverisetooneofthemostinfuriating brainteasersinthedevelopmentofmathematics:Fermat’sLastTheorem.Accordingtothetheorem,forany integervalueof n> 2,theredon’texistthreepositive integers x, y and z suchthat:

reasoning.Witheachfailedattempt,themystiqueof thetheoremgrewtohintathiddenconnectionsbetween apparentlyquitediverseareasofmathematics.

Thisdeceptivelysimplestatement,ageneralization ofthePythagoreantheorem,willbewitchmathematiciansforoverthreecenturies.Fermatpuffedthathe possesseda"trulymarvelousdemonstration,"butone cannotbecertainthatheeverdidhavesuchademonstration.Intheend,hischallengeencouragedcenturies ofmathematicianstopushthelimitsofnumbertheory.

2EarlyProgress

ThebeautyofFermat’sLastTheoremwasbuta partoftheinnocuous-soundingstatementwithwhich itfacedresistanceuntilitsproofwasfound.Itshonelike abeacon,luringbrilliantmindsdeepintomathematical

TheproofofFermat’sLastTheoremwasindeed ajourneyofwho’swhoinmathematicalhistory.The proofbroughtinnewinsightsintothesurprisingdepth andcomplexityofthistheoremonebyone.Fermat himselfprovedthecasefor n =4 byusinghismethodof infinitedescent.Thistechniqueinitseleganceandsimplicitywouldproveinfluentialinmanylaterattempts. Thecaseof n =3 wastreatedbyLeonhardEuler,the indefatigable18th-centurymathematician,whoseproof, althoughitcontainedgapsthatwerelaterfilledbyothers,showedthatthetheoremwasresistanteventothe greatestmindsoftheage.Theearly19thcenturysaw abrilliantbreakthroughbySophieGermain.Herwork onprimenumberslaidthegroundtoprovethetheorem foraninfiniteclassofprimeexponents.Germain’stheoremstandsasatestamenttoperseveranceintheface ofsocietalbarriers,assheinitiallyhadtoworkundera malepseudonymtobetakenseriouslyinthemathematicalcommunity.ErnstKummer’swork,inthemiddle ofthe19thcentury,generalizedthistothetheoryof idealnumbers,whichallowedforuniquefactorization incertainnumberfields.Thisasastrongtool,infact, enabledKummertoproveFermat’sLastTheoremfora largeclassofprimeexponentsandbroughtmathematicianstantalizinglyclosetoageneralproof.

3BreakthroughsinEllipticCurvesandModularForms

TheproofofFermat’sLastTheoremhadacompletelysurprisingbreakthrough:thetheoryofelliptic curvesandmodularforms.Thesolutionopeneddoors tocenturies-oldproblems.However,thisparticularcorrelationwastheonethatwasfirstperceivedtheother wayaround.YutakaTaniyamaandGoroShimura,two mathematiciansfromJapan,alongwithAndréWeil, putforwardafamoustheoryduringthe1950sthatcon-

nectedthetwoconcepts.Theyclaimedthatthemodularformwasaspecializedtransformationoftheelliptic formandthat a = y2 and b = x3 + ax + b

ThesevariablesbarenoresemblancetoFermat’s equationbutthefunctionalconnectioniswhatMathematicianslatercalledtheModularTheorem.Gerhard Freymadeastrongargumentin1984tobacktheclaim, statingthatifthetheoremwasproven,itwouldposeas asolutiontotheFermat’sLastTheorem.Hisevidence wasstrongasoneproblemwouldyieldmanysolutions andgivebirthtoaspecialtypeofellipticcurve.This specificcurve,knownasthemodulartype,wouldsolve manyotherequationssoifthetheoremwasprovenfalse, itwouldposeaproblemonitsown.In1986,KenRibet strengthenedthetheoryfurtherbyclaimingthatifthe ModularTheoremwasfalseforthatclassofcurves,it wouldstrengthenFrey’sargument.

EnterAndrewWiles,whosinceagetenhadbeen captivatedbyFermat’sLastTheorem.Wile’sintuitiontoldhimthatinRibet’sworktherewassomethingworthbuildingaproofoftheModularityTheoremforsemistableellipticcurves.Thenheworkedfor aboutsevenyearsinnear-secretconditions,combining techniquesthatcamefromalgebraicgeometry,number theory,andcomplexanalysisinwaysnobodyhadpreviouslydone.TheannouncementoftheproofbyWilesin 1993sentshockwaveswithinthemathematicalcommunity.Whileagapindeedwasfound,Wiles,alongwith hisformerstudentRichardTaylor,fixedthegapina year.Thefullproof,publishedin1995,provedmuch morethanFermat’sLastTheorem;itmarkedconsiderableadvancesinalgebraicnumbertheoryandarithmeticgeometry.

4Fermat’sLastTheoremandCryptography

WhileFermat’sLastTheoremitselfdoesn’thavedirectapplicationsoutsideofpuremathematics,thetools developedforitsproofdohavesurprisingusefulnessin theworldofcryptography.Thestudyofellipticcurves isattheveryheartoftheproofbyWilesandnowadaysformsthegroundforsomereallysecureencryption methods.TheideabehindECCisthatit’spublic-key cryptographythatbasesitsfoundationonthealgebraic structureofellipticcurvesoverfinitefields.

Itfindsitsresistanceonthehardnessassumptionof theECDLPproblem,whichisconsideredtobemuch intractablethantheintegerfactorizationproblem,used inthetraditionalRSAcryptography.InECC,thepublickeyisapointontheellipticcurve,whiletheprivate keyissomerandomnumber.Inotherwords,thepublickeyisobtainedbymultiplicationoftheprivatekey withageneratorpointonthecurve.Thestrengthof ECCliesinthefactthatwhileitisrelativelyeasyto performthismultiplication,it’scomputationallyinfeasibletodeterminetheprivatekeygiventhepublickey andthegeneratorpoint.TheadvantagesofECCover traditionalcryptographicsystemsareasfollows:

Smallerkeysizes,whereinforthesamestrengthof thekey,ECCcanhaveverysmallkeysizescomparedto RSA.Forinstance,a256-bitECCkeywouldhavecomparablesecurityasthatofa3027-bitRSAkey.Faster computationsduetothesmallerkeysizeresultinlow powerconsumption,whichmakesECCverysuitablefor resource-constraineddevicessuchassmartphonesand IoTdevices.

Bandwidthefficiency:

ThekeysizeandsignatureinECCaremuch smaller,reducingbandwidthusage;hence,thisisuseful overnetworkcommunication.

Thesebenefitscomewithareason,whichiswhy mostsecurityprotocolsandsystemsareusingittoday:

TransportLayerSecurityorTLS:

MostwebsitesnowuseECCaspartoftheirTLSto establishsecureHTTPS.

Bitcoinandcryptocurrencies: Bitcoinprotocolemploysthe"secp256k1"elliptic curveinitspublickeycryptography.

Fig.1:TheSecp256k1EllipticCurve

SecureShell(SSH):

OpenSSHsupportsECCforkeyexchangeanddigitalsignatures.Governmentsystems:TheNSAhasrecommendedECCtoprotectclassifiedinformation.

5Conclusion

TheresolutionofFermat’sLastTheoremisoneof thegreattriumphsofmathematicsinthe20thcentury.Notonlydiditsealapageoutofmathematics book,butdoorsfornewinvestigationswereopened,and deepconnectionsbetweenseeminglyrelativelydisparate branchesofmathematicswererevealed.

FromFermat’smarginalnotetotheproofgivenby Wiles,centuriesofmathematicaldevelopmentwereinvolvedinnumbertheory,algebraicgeometry,andcomplexanalysis.Itwasatestimonytothewaymathematicsprogressesingeneral:eachgenerationelaborateda bitfurtherwherethepreviousonehadstopped.

Theunexpecteduseofellipticcurvesinthenewer formsofcryptographyisagainmerelyafurtherinstanceoftheways-sometimesentirelyunexpected-in which‘pure’mathematicspaysitsdividend.Continue aswemayintothefrontiersofmathematics,wecanonly speculatewhatotherdeepinterconnectionsandapplicationswillbeuncovered.

Fermat’sLastTheoremisareminderofthepersistenceofmathematicalcuriosityandthefantasticinsightsthatmayresultfromwhat,atfirstglance,was apparentlyanabstractproblem.Itremainsasinspirationforbothmathematiciansandscientistsintackling, withpersistencecreativity,andaspiritofcollaboration, problemswhichmostresistsolution.

Butintheend,Fermat’sLastTheoremisatale notsomuchofamathematicalstatementbutrather ofhumaningenuity,beauty,andevenunexpectedfruits thatarisethroughthemathematicalargumentationof abstractconceptionsaboutrealityandourtechnological capabilityintheworld.Fromahumblemarginalnoteto theverybedrockofmoderncryptography,Fermat’sLast Theoremhascontinuedtobeguileandchallengeusein itsinvitationtotheendlessfrontiersofmathematical discovery.

6Bibliography

1. “WhatIsEllipticCurveCryptography?Definition FAQs.”VMware,www.vmware.com/topics/ellipticcurve-cryptography.Accessed23Jan.2025.

2. “Fermat’sLastTheorem.”Wikipedia, WikimediaFoundation,21Jan.2025, en.wikipedia.org/wiki/Fermat

3. CryptographyandFermat’sLastTheorem,ieeexplore.ieee.org/document/10315806.Accessed23 Jan.2025.

4. Broad,WilliamJ.“MathThatHelped SolveFermat’sTheoremNowSafeguards theDigitalWorld.”TheNewYorkTimes, TheNewYorkTimes,31Jan.2022, www.nytimes.com/2022/01/31/science/fermatelliptic-curves-encryption.html.

5. Dawar,Harshit.“Fermat’sTheoreminItsCore!” Medium,AnalyticsVidhya,17Oct.2020,

ArithmeticSequenceandGeometricSequencein Music

Email:shjey30@pupils.nlcsjeju.kr

RecommendedYearLevel:KS3

Keywords:Arithmeticsequence,Geometricsequence, Frequency,Octave

1Introduction

Thestructureofamelodyoftenfollowssequences, particularlyarithmeticorgeometricsequences.Forinstance,whenamelodyrepeatsorchangesatspecific intervals,thoseintervalsformasequence,whichplaysa crucialroleincontrollingthepredictabilityandmusical tensionofthemelody.

2Definition

2.1ArithmeticSequence

Anarithmeticsequenceinvolvesaseriesofnumbers whereeachtermincreasesbyaconstantdifference.In musicalterms,thisconceptisexemplifiedintheharmonicseries,whereeachovertoneisanintegermultiple ofafundamentalfrequency.

Fig.1:FundamentalFrequency

2.2GeometricSequence

Ageometricsequenceconsistsofnumberswhere eachtermismultipliedbyaconstantratio.Inmusic,thisisobservedintherelationshipbetweenpitches acrossoctaves.

Here, an representsthe n-thtermofasequence, a1 isthefirstterm, d denotesthecommondifference(forarithmeticsequences),and n indicatesthepositionoftheterminthesequence.

Thissequence(f, 2f, 3f, 4f,... )representsanarithmeticprogressionwhereeachtermincreasesbythefundamentalfrequency f .Theseharmonicscontributeto thetimbreofmusicalinstruments,enrichingthesound byaddingovertonesatintegermultiplesofthefundamentalpitch.

Here, an representsthe n-thtermofasequence, a1 isthefirstterm, r denotesthecommonratio(for geometricsequences),and n indicatesthepositionof theterminthesequence.

Ifanotehasafrequencyf,thenoteoneoctave higherhasafrequency 2 f

Twooctaveshigher: 22 f =4f

Threeoctaveshigher: 23 f =8f

Thissequence(2f, 4f, 8f,... )formsageometric progressionwithacommonratioof2.Thisexponentialrelationshipdefineshowpitchesareperceivedand organised,ensuringthateachoctavespansafrequency rangethatdoublesthepreviousone.

a

2.3InterplayBetweenArithmeticandGeometricSequences

Theinteractionbetweenthesesequencesisfundamentaltomusicalharmonyandscaleconstruction. Whiletheharmonicseries(arithmetic)definestheovertonestructureofasinglepitch,theoctaverelationships (geometric)determinehowthesepitchesarespaced acrossthemusicalspectrum.

2.3.1ExampleQuestion

Amusicianiscomposingapiecebasedona sequenceofnotesandwantstocombinearithmeticand geometricprogressionstocreateauniquemelody.The musicianstartswithanoteat 200Hz (A3).

• Eachsuccessivenoteinthe arithmeticsequence increasesby 55Hz forthefirst4notes

• Afterthe4thnote,a geometricsequence is applied,wheretheratiobetweensuccessivefrequencies is 3 2 (perfectfifthinterval)

1.Writethefrequenciesofthefirst8notesinthe sequence.

2.Determinewhetherthefinalfrequencyiswithin thesameoctaveasthestartingfrequency,assumingan octavedoublesthefrequency.

Theformulaforthe n-thtermofanarithmetic sequenceis: an = a1 +(n 1)d

• a1 =220Hz

• a2 =220+(2 1) · 55=220+55=275Hz

• a3 =220+(3 1) 55=220+110=330Hz

• a4 =220+(4 1) · 55=220+165=385Hz

Thefirst4notesare: 200Hz, 275Hz, 330Hz, 385Hz

Theformulaforthe n-thtermofageometric sequenceis: an = a1 rn 1

Here, g1 =385Hz (lastnoteofthearithmetic sequence),and r = 3 2

• g1=385Hz

• g2 =385 · ( 3 2 1)=385 · 1 5=577 5Hz

• g3 =385 ( 3 2 2)=385 2.25=866.25Hz

• g4 =385 ( 3 2 3)=385 3 375=1299 375Hz

Thefirst4notesare: 385Hz, 577 5Hz, 866 25Hz, 1299.375Hz.

Itisnotwithinthestartoctave.Anoctavedoubles thefrequency.Thestartingfrequencyis220Hz.The octaveboundariesare:

• 220Hz to 440Hz :A3toA4

• 440Hz to 880Hz :A4toA5

• 880Hz to 1760Hz :A5toA6

• 220Hz :A3(originalpitch)

• 275Hz :A3

• 330Hz :A3

• 385Hz :A3

• 577 5Hz :A4

• 866 25Hz :A4

• 1299 375Hz :A5

2.3.2ActualExamplesinMusicHistory

ClassicalExamples:Bach’s“PreludeinCMajor” featuresascendingsequencesthatbuildintensityand leadthelistenerthroughintricatemelodicdevelopment. Vivaldi’s“Winter”fromTheFourSeasonsshowcasesdescendingsequencesthatreplicatefallingsnow andenhancethethematicelementsofthepiece.

ModernExamples:TheBeatles’’LetItBe’incorporatesascendingsequencesinthepianoaccompaniment, addingemotionalupliftandreinforcingthesong’shopefulmessage.

Radiohead’s“KarmaPolice”employsadescending sequenceinthechorus,creatingasomber,contemplativemoodthatcomplementsthelyrics.

3Conclusion

Arithmeticsequencescandefinerhythmicpatterns, melodicprogressions,orharmonicstructures.Besides, geometricsequencesarefundamentalintuningsystems andpitchrelationships.Bothconceptshighlightthe profoundrelationshipbetweenmusicandmathematics, providingcomposersandperformerswithtoolstocreate structuredandharmonicallyrichmusic.

4Extension

Hereareadditionalexamplesofhowalgebraic mathematicsareappliedinmusic.Mathematicsplaysa significantroleinvariousaspectsincludingmusic,where ithelpsstructureandenrichcreativeprocesseslikemusiccompositionandanalysis.

4.1Transformation

Inmusic,transformationsallowamelodyormotiftobedevelopedandvariedwhileretainingitscore identity.Thesetransformationsareakintoalgebraicoperationsinmathematics,whereabasicformisaltered accordingtoaspecificrule.Here’sadeeperlookinto eachtypeoftransformation:

4.1.1Transposition

• Definition:Transpositioninvolvesshiftinga melodyormotifupordownbyaspecificinterval.This doesnotchangetheinternalrelationshipsbetweenthe notesbutchangestheirabsolutepitch.

• MathematicalView:Ifthemelody’snotesarerepresentedas{x1,x2,x3,... }thenatranspositionbyan intervalresultsin{x1 + k,x2 + k,x3 + k,... }

• UseinMusic:Transpositionisessentialinmodulation(keychanges),makingamotifadaptabletodifferentharmoniccontexts.

4.1.2Inversion

• Definition:Inversionflipsamelodyaroundacentralaxis,effectivelyreversingthedirectionofeachinterval.Ascendingintervalsbecomedescendingonesand viceversa.

• MathematicalView:Ifthemelody’snotesare {x1,x2,x3,... }inversiontransformsthemto{c x1,c x2,c x3,... }wherecistheaxisofsymmetry.

• UseinMusic:Inversioncreatescontrastwhile maintainingaconnectiontotheoriginaltheme.It’s commonincounterpointandfugues(e.g.,J.S.Bach).

4.2Symmetry

Symmetryinmusicreferstothebalanceandproportioncreatedwhenonesectionofamelodymirrors orcomplementsanother.Thiscanbeachievedthrough reflectiveorrotationalsymmetry.

4.2.1ReflectiveSymmetry

• Definition:Reflectivesymmetryoccurswhenone partofamelodymirrorsanotherpartacrossacentral axis.

• MathematicalView:Ifthemelody’snotes are{x1,x2,x3,... }reflectivesymmetrycreatesanew melody{xn,xn 1,xn 2,... }

• UseinMusic:Reflectivesymmetryenhancesbalance,oftenusedincompositionslikepalindromicstructures(e.g.,Bartók’s"MusicforStrings,Percussion,and Celesta").

4.2.2RotationalSymmetry

• Definition:Rotationalsymmetryinvolvesrepeatingamelodyormotifincycles,butstartingatdifferent points,similartorotatingashapeingeometry.

• UseinMusic:Rotationalsymmetrycreatesvariationwhilepreservingthecoreidentityofthemelody, commonlyseeninminimalism(e.g.,SteveReich’s works).

5Bibiliography

1. CMUSE."ClassicalPieceswiththeGoldenRatio." CMUSE,www.cmuse.org/classical-pieces-with-thegolden-ratio.

2. ProMusicianHub."WhatIsSequenceinMusic?" ProMusicianHub,promusicianhub.com/what-issequence-music/.

3. AudioApartment."SequenceinMusic:MusicTheoryandComposition."AudioApartment,audioapartment.com/music-theory-andcomposition/sequence-in-music/.

4. Fiveable."Motive,MotivicDevelopment,andPitchTransformation."Fiveable,library.fiveable.me/ap-music-theory/unit6/motive-motivic-pit-transformation/studyguide/z0DJQvgjoByphnhSnztH.

5. Vaia."MusicalSymmetry:Music Composition."Vaia,www.vaia.com/enus/explanations/music/musiccomposition/musical-symmetry/.

Volume5,Issue1

MathematicsSociety MathematicsPublicationCCA

April2025

Turn static files into dynamic content formats.

Create a flipbook
Issuu converts static files into: digital portfolios, online yearbooks, online catalogs, digital photo albums and more. Sign up and create your flipbook.
The NLCS Jeju Journal of Pure and Applied Mathematics Volume 5, April 2025 by North London Collegiate School Jeju - Issuu